N316B Midterm 1

Pataasin ang iyong marka sa homework at exams ngayon gamit ang Quizwiz!

Which client statement indicates understanding of the side effects of nitroglycerin ointment? 'I may experience a headache.' 'Confusion is a common adverse effect.' 'A slow pulse rate in an expected side effect.' 'Increased blood pressure readings may occur initially.'

*'I may experience a headache.' Rationale The most common side effect of nitroglycerin is a headache. Additional cardiovascular side effects are hypotension, not hypertension; tachycardia, not bradycardia; and dizziness, not confusion.

An 80-year-old client with depression requires the prescription of antidepressant medication. Which tricyclic antidepressant medication causes fewer complications in older clients? -Doxepin -Amoxapine -Nortriptyline -Trimipramine

-Nortriptyline Rationale: Nortriptyline and desipramine are preferred for use in older adults because these antidepressant medications have less anticholinergic activity. Doxepin, amoxapine, and trimipramine have more cholinergic activity than nortriptyline and are not the preferred medications for older adult clients.

687) A client taking multiple medications for hypertension develops a persistent, hacking cough. Which antihypertensive medication class would the nurse identify as the likely cause of the cough? Thiazide diuretics Calcium channel blockers Direct renin inhibitors Angiotensin-converting enzyme (ACE) inhibitors

Angiotensin-converting enzyme (ACE) inhibitors Rationale The ACE breaks down kinins. When ACE is inhibited, the increase of kinins in the lung can cause bronchial irritation, leading to the common adverse effect sometimes referred to as an ACE cough. A cough is not a side effect of thiazide diuretics, calcium channel blockers, or direct renin inhibitors.

List in order the steps the nurse teaches the client to follow when using a metered-dose inhaler (MDI). 1. Shake the inhaler for 30 seconds. 2. Exhale slowly and deeply to empty the air from the lungs. 3. Hold the inhaler upright in the mouth. 4. Start breathing in and press down on the inhaler once.

Answer: 1,2,3,4,5 Rationale When using an MDI, the medication should be shaken for 30 seconds to ensure that the medication is mixed. Exhaling completely maximizes emptying the lungs. The inhaler should be held upright in the mouth past the teeth with the lips closed around the mouthpiece (closed mouth method) or held upright 1 to 2 cm in front of the open mouth (open mouth method). Inhalation is begun at the same time that the device is compressed to ensure that maximum medication reaches the lungs.

The healthcare provider prescribes digitalis (Digoxin) for a client diagnosed with heart failure. Which intervention should the nurse implement prior to administering the digoxin? A. Observe respiratory rate and depth. B. Assess the serum potassium level. C. Obtain the client's blood pressure. D. Monitor the serum glucose level.

B. Assess the serum potassium level. Hypokalemia (decreased serum potassium) will precipitate digitalis toxicity in persons receiving digoxin. The nurse should monitor the client's serum potassium levels. Blood pressure and respiratory rate will not inform the nurse about potential safety issues with digitalis.

The nitrate isosorbide dinitrate is prescribed for a client with angina. Which instruction should the nurse include in this client's discharge teaching plan? A. Quit taking the medication if dizziness occurs. B. Do not get up quickly. Always rise slowly. C. Take the medication with food only. D. Increase your intake of potassium-rich foods.

B. Do not get up quickly. Always rise slowly. An expected side effect of nitrates is orthostatic hypotension and the nurse should instruct the client to prevent it by rising slowly.

A client with Parkinson's disease is taking carbidopa-levodopa (Sinemet). Which observation by the nurse would indicate that the desired outcome of the medication is being achieved? A. Decreased blood pressure. B. Lessening of tremors. C. Increased salivation. D. Increased attention span.

B. Lessening of tremors. Rationale: Sinemet increases the amount of levodopa to the CNS (dopamine to the brain). Increased amounts of dopamine improve the symptoms of Parkinson's, such as involuntary movements, resting tremors, shuffling gait, etc. Decreased drooling would be a desired effect, not increased salivation.

A client being discharged is prescribed warfarin for the treatment following a pulmonary embolism. Which diagnostic test should the nurse instruct the client to receive once a month? A. Perfusion scan. B. Prothrombin Time (PT). C. Activated partial thromboplastin (aPTT). D. Serum Coumadin level (SCL).

B. Prothrombin Time (PT). When used for a client with pulmonary embolus, the therapeutic goal for warfarin therapy is a PT 1 to 2 times greater than the control, or an INR of 2 to 3. A client prescribed warfarin should have the PT or INR levels checked at a minimum once a month.

The nurse is providing care for a client prescribed propranolol. Which symptoms should the nurse report to the healthcare provider immediately? A. Headache, hypertension, and blurred vision. B. Wheezing, hypotension, and AV block. C. Vomiting, dilated pupils, and papilledema. D. Tinnitus, muscle weakness, and tachypnea.

B. Wheezing, hypotension, and AV block. Wheezing, hypotension, and AV block represents the most serious adverse effects of beta-blocking agents. AV block is generally associated with bradycardia and results in potentially life-threatening decreases in cardiac output. Additionally, wheezing secondary to bronchospasm and hypotension represent life-threatening respiratory and cardiac disorders.

The nurse is teaching a pediatric client and family about prescribed albuterol sulfate extended-release tablets. Which statement should be included? AIf you cannot swallow the tablet, it is ok to chew it BThis medication can cause restlessness CRinse your mouth after taking this medication DOral albuterol can cause an increase in urination

BThis medication can cause restlessness Rationale: The adverse reactions to albuterol are the same whether administered orally or via inhalation. The most frequent adverse reactions to albuterol are nervousness, tremors, headache, tachycardia, and palpitations. Less frequent adverse reactions are muscle cramps, insomnia, nausea, weakness, dizziness, drowsiness, flushing, restlessness, irritability, chest discomfort, and difficulty in urination. Extended-release medications should not be chewed or crushed. Doing so can release all of the drug at once, increasing the risk of side effects. Inhaled corticosteroids require the mouth to be rinsed. This medication is not inhaled and is not a corticosteroid.

The nurse is reinforcing medication interactions with a client who is taking warfarin. Which over-the-counter (OTC) medication should the nurse remind the client to avoid? ADiphenhydramine BAcetaminophen CNaproxen DPantoprazole

CNaproxen Rationale: Warfarin is an anticoagulant. OTC medications that interact with warfarin should be avoided. Naproxen, a nonsteroidal anti-inflammatory drug (NSAID), is a commonly used OTC analgesic. Naproxen can prolong bleeding time and should therefore be avoided by clients who take anticoagulants. The other medications are not contraindicated when taking warfarin.

The nurse is teaching a client diagnosed with depression about a new prescription of nortriptyline. What information would be essential for the nurse to emphasize about this medication? AEpisodes of diarrhea can be expected BThe medication must be stored in the refrigerator CThe use of alcohol should be avoided DSymptom relief occurs in a few days

CThe use of alcohol should be avoided Rationale: Nortriptyline is a tricyclic antidepressant used to manage chronic neurogenic pain and depression. Adverse reactions include central nervous system (CNS) side effects such as suicidal thoughts, drowsiness, fatigue, lethargy, and confusion. Clients who are prescribed this medication should be educated to avoid the use of alcohol consumption or other CNS depressant drugs as this can worsen the adverse reactions of the medication and cause injury.

The nurse administers a parenteral preparation of potassium slowly to avoid which complication? Metabolic acidosis Cardiac arrest Seizure activity Respiratory depression

Cardiac arrest Rationale Too rapid an administration can cause hyperkalemia, which contributes to a long refractory period in the cardiac cycle, resulting in cardiac dysrhythmias and arrest. Although acidosis can cause hyperkalemia, hyperkalemia will not lead to acidosis. Hyperkalemia causes muscle flaccidity and weakness, not seizures. Respiratory depression can occur with too rapid intravenous (IV) magnesium administration, not potassium administration.

Which drink would a nurse teach a client on warfarin to avoid? Apple juice Grape juice Orange juice Cranberry juice

Cranberry juice Rationale The antioxidants in cranberry juice may inhibit the mechanism that metabolizes warfarin, causing elevations in the international normalized ratio, resulting in hemorrhage. Apple juice, grape juice, and orange juice are fine to drink.

A client with coronary artery disease who is taking digoxin (Lanoxin) receives a new prescription for atorvastatin (Lipitor). Two weeks after initiation of the Lipitor prescription, the nurse assesses the client. Which finding requires the most immediate intervention? A. Heartburn. B. Headache. C. Constipation. D. Vomiting.

D. Vomiting. Vomiting, anorexia, and abdominal pain are early indications of digitalis toxicity. Since Lipitor increases the risk for digitalis toxicity, this finding requires the most immediate intervention by the nurse

Which therapeutic effect would the nurse expect to identify when mannitol is administered to a client? Improved renal blood flow Decreased intracranial pressure Maintenance of circulatory volume Prevention of the development of thrombi

Decreased intracranial pressure Rationale As an osmotic diuretic, mannitol helps reduce cerebral edema. Although there may be a transient increase in blood volume as a result of an increased osmotic pressure, which increases renal perfusion, this is not the therapeutic effect. Prevention of the development of thrombi is not the reason for giving this medication.

A beclomethasone inhaler would be prescribed for which purpose? Prevents atelectasis Decreases inflammation Relaxes smooth muscle in the airways Reduces bacteria in the respiratory tract

Decreases inflammation Rationale Beclomethasone reduces the inflammatory response in bronchial walls by suppression of polymorphonuclear leukocytes and fibroblasts and the reversal of capillary permeability. Beclomethasone does not prevent atelectasis. Beclomethasone does not cause smooth muscle relaxation in the airways. Beclomethasone is not an antibiotic.

886) A client with left ventricular heart failure and supraventricular tachycardia is prescribed digoxin 0.25 mg daily. Which changes would the nurse expect to find if this medication is therapeutically effective? Select all that apply. One, some, or all responses may be correct. Diuresis Tachycardia Decreased edema Decreased pulse rate Reduced heart murmur Jugular vein distention

Diuresis Decreased edema Decreased pulse rate Rationale Digoxin increases kidney perfusion, which results in urine formation and diuresis. The urine output increases because of improved cardiac output and kidney perfusion, resulting in a reduction in edema. Because of digoxin's inotropic and chronotropic effects, the heart rate will decrease. Digoxin increases the force of contractions (inotropic effect) and decreases the heart rate (chronotropic effect). Digoxin does not affect a heart murmur. Jugular vein distention is a specific sign of right ventricular heart failure; it is treated with diuretics to reduce the intravascular volume and venous pressure.

The nurse is administering hydroxyzine to a client. The nurse would monitor the client for which side effect of this medication? Ataxia Drowsiness Vertigo Slurred speech

Drowsiness Rationale Hydroxyzine suppresses activity in key regions of the subcortical area of the central nervous system; it also has antihistaminic and anticholinergic effects. Ataxia, vertigo and slurred speech are not associated with hydroxyzine.

The nurse is counseling a 34-year-old client who has requested a prescription for oral contraceptives. Which condition would warrant additional discussion? Anemia Depression Hypertension Dysmenorrhea

Hypertension Rationale One of the side effects of oral contraceptives is hypertension; therefore they are contraindicated for any woman who already has hypertension, particularly at the client's age or older. Anemia is not a contraindication for women who want to take oral contraceptives because oral contraceptives may help this condition by decreasing bleeding. Depression is not a contraindication for women who want to take oral contraceptives. Oral contraceptives may be prescribed for women with menstrual difficulties such as dysmenorrhea.

A client is admitted to the hospital for a new onset of supraventricular tachycardia (SVT) and is prescribed digoxin. For which laboratory finding should the nurse notify the healthcare provider immediately? A. Potassium level of 3.1 mEq/L. B. Sodium level of 132 mEq/L. C. Calcium level of 8.6 mg/dL. D. Magnesium level of 1.2 mEq/L.

Hypokalemia affects myocardial contractility and places this client at greatest risk for dysrhythmias that may be unresponsive to drug therapy. Although an imbalance of serum sodium, calcium, and magnesium can effect cardiac rhythm, the greatest risk for a client receiving digoxin is low potassium.

A client presents to the emergency department with chest pain. A myocardial infarction is suspected, and 500 mL of 5% dextrose in water (D 5W) with 50 mg of nitroglycerin intravenously (IV) has been prescribed. The nurse will monitor the client for which common side effect of nitroglycerin? Bradycardia Hypotension Nausea and vomiting Leg cramps

Hypotension Rationale The major action of intravenous nitroglycerin is venous and then arterial dilation, leading to a decrease in blood pressure; orthostatic hypotension can occur. Bradycardia is not an anticipated response. Nausea and vomiting may occur but are not the most common side effects of IV nitroglycerin. Leg cramps are not a side effect of this medication.

A 6-year-old child is receiving an intravenous solution of 10% glucose and mannitol to reduce cerebral edema. Which complication would the nurse monitor the child for? Overhydration Seizure activity Acute heart failure Hypovolemic shock

Hypovolemic shock Rationale Both hypertonic glucose and mannitol cause diuresis; the child should be monitored for excessive fluid loss. Hypertonic glucose and mannitol will cause fluid loss, not gain. Seizure activity is not anticipated as a result of this infusion. An increased fluid volume can lead to heart failure; however, hypertonic glucose and mannitol cause fluid loss, not gain.

Which times for the medication schedule would a nurse teach when corticosteroid therapy is prescribed for a client with an exacerbation of ulcerative colitis? At bedtime with a snack Three times a day with meals In the early morning with food One hour before or 2 hours after eating

In the early morning with food Rationale Taking the medication in the early morning mimics usual adrenal secretions; food helps reduce gastric irritation. Diurnal rhythms may be altered, and steroids are ulcerogenic; they should be taken with more than just a snack. Steroids cause gastric irritation and should be taken with food. Although food helps decrease gastric irritation, dividing the dose and taking it throughout the day may alter regular diurnal rhythms; it should be taken in the early morning with food.

A client is receiving dexamethasone to treat acute exacerbation of asthma. For which side effect would the nurse monitor the client? Hyperkalemia Liver dysfunction Orthostatic hypotension Increased blood glucose

Increased blood glucose Rationale Dexamethasone increases gluconeogenesis, which may cause hyperglycemia. Hypokalemia, not hyperkalemia, is a side effect. Liver dysfunction is not a side effect. Hypertension, not hypotension, is a side effect.

The nurse is teaching the parents of a child prescribed a high dose of oral prednisone for asthma. Which information is critical for the nurse to include when teaching about this medication? It protects against infection. It should be stopped gradually. An early growth spurt may occur. A moon-shaped face will develop.

It should be stopped gradually. Rationale Gradual weaning from prednisone is necessary to prevent adrenal insufficiency or adrenal crisis. Prednisone depresses the immune system, thereby increasing susceptibility to infection. The medication usually suppresses growth. A moon face may occur, but it is not a critical, life-threatening side effect.

Which side effect would the nurse assess for in a child receiving prednisone? Alopecia Anorexia Weight loss Mood changes

Mood changes Rationale Mood swings may result from steroid therapy. Alopecia does not result from steroid therapy. An increased appetite, not anorexia, results from steroid therapy. Weight gain, not weight loss, results from steroid therapy.

A client who has type 1 diabetes and chronic bronchitis is prescribed atenolol for the management of angina pectoris. Which clinical manifestation will alert the nurse to the fact that the client may be developing a life-threatening response to the medication? Paroxysmal nocturnal dyspnea Supraventricular tachycardia Malignant hypertension Hyperglycemia

Paroxysmal nocturnal dyspnea Rationale Atenolol is associated with the adverse reactions of bradycardia, heart failure, and pulmonary edema; these are the most serious responses to atenolol and are often manifested by episodes of paroxysmal nocturnal dyspnea and orthopnea. A decreased, not increased, pulse rate is associated with atenolol so supraventricular tachycardia is not a response. Atenolol decreases, not increases, blood pressure so malignant hypertension is not a response. It also will not cause an increase in blood glucose. It may increase the hypoglycemic response to insulin, causing hypoglycemia. In addition, the medication may mask the clinical manifestations of hypoglycemia.

A pediatric client is prescribed an intravenous infusion of methylprednisolone. Which clinical manifestation requires immediate intervention during administration of the initial dose? Polyuria Tinnitus Drowsiness Hypotension

Polyuria Rationale Intravenous administration of a steroid can cause a rapid increase in the blood glucose level. One early sign of hyperglycemia is increased urine output. Blood glucose should be checked frequently, and insulin administered as needed. Tinnitus is associated with some antibiotics and with aspirin, not steroids. Drowsiness is associated with sedatives, not steroids. Hypertension, not hypotension, is associated with steroid administration.

Which medication is unsafe to administer as an intravenous (IV) bolus? Saline flush Potassium chloride Naloxone Adenosine

Potassium chloride Rationale Potassium chloride given as an IV bolus can cause cardiac arrest. It must be diluted and infused slowly through an IV infusion pump. Saline flush, naloxone, and adenosine are appropriate to be given as an IV bolus undiluted.

Which dietary choices will the nurse instruct the client taking spironolactone to avoid increasing? Select all that apply. One, some, or all responses may be correct. Potatoes Red meat Cantaloupe Wheat bread Flavored yogurt

Potatoes Cantaloupe Rationale Spironolactone is potassium-sparing, and beverages and foods containing potassium such as potatoes, cantaloupe, bananas, avocados, oranges, dates, apricots, and raisins should not be increased beyond the client's ordinary consumption to prevent hyperkalemia. Red meat may need to be limited for other reasons not related to spironolactone. Whole grains are associated with prevention of constipation and should not be avoided. Dairy products are rich in sodium and calcium; spironolactone may cause hyponatremia.

Which substance does vitamin K contributes to the formation of? Bilirubin Prothrombin Thromboplastin Cholecystokinin

Prothrombin Rationale Vitamin K is necessary in the formation of prothrombin to prevent bleeding. It is a fat-soluble vitamin and is not absorbed from the gastrointestinal (GI) tract in the absence of bile. Bilirubin is the bile pigment formed by the breakdown of erythrocytes. Thromboplastin converts prothrombin into thrombin during the process of coagulation. Cholecystokinin is the hormone that stimulates contraction of the gallbladder

A 25-year-old woman on estrogen therapy has a history of smoking. Which complication would the nurse anticipate in the client? Osteoporosis Hypermenorrhea Endometrial cancer Pulmonary embolism

Pulmonary embolism Rationale Estrogen therapy increases the risk of pulmonary embolism in clients who have a history of smoking because the medication affects blood circulation and hemostasis. Osteoporosis may be caused by reduced bone density observed in postmenopausal woman. Hypermenorrhea (excessive menstrual bleeding) is treated with estrogen therapy. Endometrial cancer is a complication of estrogen therapy seen in postmenopausal woman.

A client with rheumatoid arthritis asks the nurse why it is necessary to inject hydrocortisone into the knee joint. Which reason would the nurse include in a response to this question? Lubricates the joint Reduces inflammation Provides physiotherapy Prevents ankylosis of the joint

Reduces inflammation Rationale Steroids have an anti-inflammatory effect that can reduce arthritic pannus formation. Injecting hydrocortisone into the joint does not provide lubrication. Injection of a medication into a joint is not physiotherapy. Ankylosis refers to fusion of joints. It is only indirectly influenced by steroids, which exert their major effect on the inflammatory process.

The nurse is monitoring a client who is taking prescribed nitroglycerin for angina. Which finding indicates the medication has a therapeutic effect? AThe client blood pressure is 150/80 mm/Hg. BThe client heart rate is 110. CThe client reports a decrease in chest pressure. DThe client reports a headache.

Rationale: Nitroglycerin acts to decrease myocardial oxygen consumption. Dilatation of the veins reduces the amount of blood returning to the heart (preload), so the chambers have a smaller volume to pump resulting in decreased oxygen needs. Decreased oxygen demand reduces pain caused by dilating coronary blood flow. While blood pressure may decrease slightly due to the vasodilatory effects of nitroglycerin, it is a secondary effect and not the desired therapeutic effect of this drug. Increased blood pressure and increased preload would mean the heart would work harder, increasing oxygen demand and thus angina. Decreased heart rate is not an effect of nitroglycerin.

Which medication requires the nurse to monitor the client for signs of hyperkalemia? Furosemide Metolazone Spironolactone Hydrochlorothiazide

Spironolactone Rationale Spironolactone is a potassium-sparing diuretic; hyperkalemia is an adverse effect. Furosemide, metolazone, and hydrochlorothiazide generally cause hypokalemia.

597) The nurse administers albuterol to a child with asthma. Which common side effect would the nurse monitor for in the child? Flushing Dyspnea Tachycardia Hypotension

Tachycardia Rationale Albuterol produces sympathetic nervous system side effects such as tachycardia and hypertension. Pallor, not flushing, is a common side effect. Dyspnea is not a common side effect; this medication is given to decrease respiratory difficulty. Hypertension, not hypotension, is a common side effect.

A health care provider prescribes metaproterenol for a client. For which therapeutic effect would the nurse monitor the client? Induced sedation Relaxed bronchial spasm Decreased blood pressure Productive cough

Relaxed bronchial spasm Rationale Metaproterenol stimulates beta receptors of the sympathetic nervous system, causing bronchodilation and an increased rate and strength of cardiac contractions. Barbiturates and hypnotics produce sedation. Antihypertensives and diuretics help decrease blood pressure. Expectorants mobilize respiratory secretions, promoting a productive cough.

Which clinical indicator would the nurse monitor to determine if the client's simvastatin is effective? Heart rate Triglycerides Blood pressure International normalized ratio (INR)

Triglycerides Rationale Therapeutic effects of simvastatin include decreased levels of serum triglycerides, low-density lipoprotein (LDL), and cholesterol. INR is not related to simvastatin; it is a measure used to evaluate blood coagulation. Heart rate and blood pressure are not related to simvastatin.

A child undergoing prolonged steroid therapy takes on a cushingoid appearance. The nurse would expect to find which of these manifestations during further assessment? Select all that apply. One, some, or all responses may be correct. Truncal obesity Thin extremities Increased linear growth Loss of hair on the body Decreased blood pressure

Truncal obesity Thin extremities Rationale An increase in appetite results in deposition of fat on the abdomen and trunk. Muscle wasting results in thin extremities. Increased excretion of calcium causes retardation of linear growth and a resulting short stature. Because of the excess production of androgens, virilization and hirsutism occur. Increased salt and water retention cause hypertension and hypernatremia.

Which assessment would the nurse perform specific to the safe administration of intravenous mannitol? Body weight daily Urine output hourly Vital signs every 2 hours Level of consciousness every 8 hours

Urine output hourly Rationale Mannitol, an osmotic diuretic, increases the intravascular volume that must be excreted by the kidneys. The client's urine output should be monitored hourly to determine the client's response to therapy. Although mannitol results in an increase in urinary excretion that is reflected in a decrease in body weight (1 L of fluid is equal to 2.2 pounds [1 kg]), a daily assessment of the client's weight is too infrequent to assess the client's response to therapy. Urine output can be monitored hourly and is a more frequent, accurate, and efficient assessment than is a daily weight. Vital signs should be monitored every hour considering the severity of the client's injury and the administration of mannitol. Although the level of consciousness should be monitored with a head injury, conducting assessments every 8 hours is too infrequent to monitor the client's response to therapy.

Which prescription would the nurse anticipate for the client who takes a medication that interferes with fat absorption? High-fat diet Supplemental cod liver oil Total parenteral nutrition (TPN) Water-miscible forms of vitamins A and E

Water-miscible forms of vitamins A and E Rationale Vitamins A, D, E, and K are known as fat-soluble vitamins because bile salts and other fat-related compounds aid their absorption. A high-fat diet will not achieve the uptake of fat-soluble vitamins in this client. Supplemental cod liver oil will not achieve the uptake of fat-soluble vitamins in this client. TPN is unnecessary; a well-balanced diet is preferred. Water-miscible forms of vitamins A and E can be absorbed with water-soluble nutrients.

Which side effect of prolonged cortisone therapy for adrenal insufficiency would the nurse teach the client and family to expect? Select all that apply. One, some, or all responses may be correct. Oliguria Anorexia Weakness Moon face Weight gain Nervousness

Weakness Moon face Weight gain Rationale Weakness occurs because of muscle wasting as a result of the catabolic effects of cortisol. Hypokalemia may also cause weakness; potassium is lost in the urine as sodium is retained. Accumulation of adipose tissue occurs in the face (moon face), trunk (truncal obesity), and cervical area (buffalo hump). Weight gain occurs because of increased appetite and fluid retention; 1 liter of fluid is equal to 2.2 pounds (1 kilogram). Cortisone increases sodium and water retention but does not cause oliguria; glucose levels also increase, which, if extreme, will cause polyuria. The appetite usually increases, not decreases. Cortisone increases blood glucose levels, which, if extreme, will cause lethargy, not nervousness.

A client has been prescribed alendronate for osteoporosis. Which statements indicate that the client understands how to safely take this medication? Select all that apply. "I will notify my doctor if I experience worsening heartburn." "I will take the pill with an antacid to prevent stomach upset." "I will swallow the pill with a full glass of water." "I will stand or sit quietly for 30 minutes after taking the pill." "I will always eat breakfast before taking the pill."

"I will notify my doctor if I experience worsening heartburn." "I will swallow the pill with a full glass of water." "I will stand or sit quietly for 30 minutes after taking the pill." Rationale: Alendronate is a bisphosphonate used to treat osteoporosis. It can cause esophagitis or esophageal ulcers unless precautions are followed. The client must sit upright or stand for at least 30 minutes after taking the medication. The client should take the medication with a full glass of water, at least 30 minutes before eating or drinking anything or taking any other medication. Antacids will interfere with absorption and should not be taken at the same time.

The nurse prepares discharge instructions for a client who will take enalapril for hypertension. Which instruction would the nurse include in the client's teaching? 'Change to a standing position slowly.' 'This may color your urine green.' 'The medication may cause a sore throat for the first few days.' 'Schedule blood tests weekly for the first 2 months.'

'Change to a standing position slowly.' Rationale Enalapril is classified as an angiotensin-converting enzyme (ACE) inhibitor. Like many antihypertensives, it can cause orthostatic hypotension. Clients should be advised to change positions slowly to minimize this effect. This medication does not alter the color of urine or cause a sore throat the first few days of treatment. Presently, there are no guidelines that suggest blood tests are required weekly for the first 2 months.

The nurse is teaching a client diagnosed with asthma about the medication albuterol. Which statement by the nurse demonstrates appropriate teaching? A"Call your doctor's office if you need to use the drug more often." B"Use this medication at bedtime to promote rest." C"Use this medication after other asthma inhalers." D"Discontinue the inhaler if you feel dizzy."

A"Call your doctor's office if you need to use the drug more often." Rationale: Albuterol is a bronchodilator used for the relief of bronchospasm. It is considered a rescue medication for a client during an asthmatic attack. If the client notices the need to use the inhaler more frequently, the health care provider (HCP) should be notified. The client may need to seek emergency medical care, as the medication is no longer effective. In addition, clients should not exceed the recommended dosage, as adverse effects may occur. Be sure the client understands how to correctly use this medication. The client may experience side effects of dizziness, headache, nausea, vomiting, rapid heart rate, anxiety, sweating, flushing and insomnia. Using albuterol at bedtime may lead to insomnia. Albuterol should be used before all other inhalers, as it dilates the bronchi or bronchioles and allows more of the other medication to reach the lower respiratory tract. It would not be appropriate to suddenly discontinue taking a bronchodilator.

The nurse is teaching a school-aged child and family members about the use of inhalers prescribed for asthma. Which statement made by a family member indicates an understanding of the nurse's instructions? A"We will keep a chart of daily peak flow meter results." B"We can rely on our child's self-report of symptoms." C"Monitoring our child's pulse rate is not necessary." D"Skin color changes in our child is an early warning sign for airway constriction."

A"We will keep a chart of daily peak flow meter results." Rationale: The peak flow meter can help determine if the symptoms of asthma are in control or are worsening. It works by measuring how fast air comes out of the lungs when the client forcefully exhales (the peak expiatory flow or PEF). The client should record the highest of three readings in an asthma diary daily. Children ages 4 and up should be able to use a peak flow meter. A decrease in PEF is an early warning sign for airway constriction and should be immediately addressed. Family members should monitor the child's pulse rate and changes in skin color is a late sign.

A client with acute myocardial infarction is admitted to the coronary care unit. Which medication should the nurse administer to lessen the workload of the heart by decreasing the cardiac preload and afterload? A. Nitroglycerin. B. Propranolol (Inderal). C. Morphine. D. Captopril (Capoten).

A. Nitroglycerin. Nitroglycerin is a nitrate that causes peripheral vasodilation and decreases contractility, thereby decreasing both preload and afterload.

Following the administration of sublingual nitroglycerin to a client experiencing an acute anginal attack, which assessment finding indicates to the nurse that the desired effect has been achieved? A. Client states chest pain is relieved. B. Client's pulse decreases from 120 to 90. C. Client's systolic blood pressure decreases from 180 to 90. D. Client's SaO2 level increases from 92% to 96%.

A. Client states chest pain is relieved. Nitroglycerin reduces myocardial oxygen consumption which decreases ischemia and reduces chest pain.

A client is being treated for osteoporosis with alendronate (Fosamax), and the nurse has completed discharge teaching regarding medication administration. Which morning schedule would indicate to the nurse that the client teaching has been effective? A. Take medication, go for a 30 minute morning walk, then eat breakfast. B. Take medication, rest in bed for 30 minutes, eat breakfast, go for morning walk. C. Take medication with breakfast, then take a 30 minute morning walk. D. Go for a 30 minute morning walk, eat breakfast, then take medication.

A. Take medication, go for a 30 minute morning walk, then eat breakfast. Alendronate (Fosamax) is best absorbed when taken thirty minutes before eating in the morning. The client should also be advised to remain in an upright position for at least thirty minutes after taking the medication to reduce the risk of esophageal reflux and irritation.

After abdominal surgery, a client is prescribed low molecular weight heparin (LMWH). During administration of the medication, the client asks the nurse the reason for the medication. Which is the best response for the nurse to provide the client? A. This medication is given to prevent blood clot formation. B. This medication enhances antibiotics to prevent infection. C. This medication dissolves clots that develop in the legs. D. This medication enhances the healing of wounds.

A. This medication is given to prevent blood clot formation. Unfractionated heparin or low molecular weight heparin (LMWH) is an anticoagulant that inhibits thrombin-mediated conversion of fibrinogen to fibrin and is given prophylactically to prevent postoperative venous thrombosis in order to prevent pulmonary embolism or deep vein thrombosis following knee and abdominal surgeries.

The nurse is preparing the 0900 dose of losartan (Cozaar), an angiotensin II receptor blocker (ARB), for a client with hypertension and heart failure. The nurse reviews the client's laboratory results and notes that the client's serum potassium level is 5.9 mEq/L. Which action should the nurse take first? A. Withhold the scheduled dose. B. Check the client's apical pulse. C. Notify the healthcare provider. D. Repeat the serum potassium level.

A. Withhold the scheduled dose. The nurse should first withhold the scheduled dose of Cozaar because the client is hyperkalemic (normal range 3.5 to 5 mEq/l). Although hypokalemia is usually associated with diuretic therapy in heart failure, hyperkalemia is associated with several heart failure medications, including ARBs. Because hyperkalemia may lead to cardiac dysrhythmias, the nurse should check the apical pulse for rate and rhythm, and blood pressure.

A nurse is teaching an 80-year-old client how to use a metered dose inhaler. The nurse is concerned that the client is unable to coordinate the release of the medication during the inhalation phase. Which intervention should improve the delivery of the medication? AAsk a family member to assist the client with the inhaler. BRequest a home health nurse to visit the client at home. CUse nebulized treatments at home instead. DAdd a spacer device to the inhaler canister.

AAsk a family member to assist the client with the inhaler. DAdd a spacer device to the inhaler canister. Rationale: Use of a spacer is especially useful with older adults because it allows more time to inhale and requires less eye-hand coordination. If the client is not using the metered dose inhaler (MDI) properly, the medication can get trapped in the upper airway and lead to dry mouth and throat irritation. Using a spacer will allow more drug to be deposited in the lungs and less in the mouth.

Propranolol is prescribed for a client with coronary artery disease (CAD). The nurse should consult with the health care provider (HCP) before giving this medication when the client reports a history of which condition? AAsthma BDeep vein thrombosis CMyocardial infarction DPeptic ulcer disease

AAsthma Rationale: Non-cardioselective beta-blockers such as propranolol block b1- and b2-adrenergic receptors and can cause bronchospasm, especially in clients with a history of asthma. Beta-blockers will have no effect on the client's peptic ulcer disease or risk for DVT. Beta-blocker therapy is recommended after an MI.

The nurse is caring for a client who is receiving a continuous intravenous heparin infusion. The client's most recent activated partial thromboplastin time (aPTT) is 120 seconds. Which medication should the nurse plan to administer? AProtamine BNaloxone CVitamin K DEnoxaparin

AProtamine Rationale: The client's aPTT is much higher than the typical desired therapeutic range of 1.5-2.5 the control value and places the client at great risk for uncontrolled bleeding. Protamine sulfate is the medication used to reverse the effects of heparin; it is a heparin antagonist. Neutralization of heparin occurs immediately and lasts for 2 hours, after which additional protamine may be needed. Protamine is administered by slow IV injection (no faster than 20 mg/ min or 50 mg in 10 minutes). Dosage is based on the fact that 1 mg of protamine will inactivate approx. 100 units of heparin. Vitamin K is used to reverse the effects of warfarin. Naloxone is used to reverse the effects of opioids. Enoxaparin is another anticoagulant (low molecular weight heparin). Question 3

The international normalized ratio (INR) results of a client receiving warfarin have been variable. Which factor can help the nurse identify the cause of the INR fluctuations? Intake of foods high in potassium Serum glucose level Platelet count Adherence to the prescribed medication regimen

Adherence to the prescribed medication regimen Rationale The dosage of warfarin is adjusted according to INR results; if the client fails to take the medication as prescribed, test results will not be reliable in monitoring the client's response to therapy. Intake of foods high in vitamin K, not potassium, is important to ascertain. Low platelet counts increase the risk for bleeding; however, they do not affect warfarin activity or the INR. Serum glucose levels do not affect the warfarin or the INR.

The nurse is caring for a client who is experiencing excessive bleeding after receiving unfractionated heparin sodium. Which orders should the nurse anticipate from the health care provider? Select all that apply. Administer vitamin K. Obtain prothrombin time (PT)/international normalized ratio (INR). Administer protamine sulfate. Obtain activated partial thromboplastin time (aPTT). Change prescription to enoxaparin.

Administer protamine sulfate. Obtain activated partial thromboplastin time (aPTT). Rationale: Protamine sulfate is the antidote used to reverse the anticoagulant effects of heparin. A serum aPTT or PTT lab test is used to evaluate the anticoagulation effect of heparin. Vitamin K is the antidote for warfarin. A serum PT/INR lab test is used to monitor the therapeutic effectiveness of warfarin. Enoxaparin is another type of heparin and would be contraindicated for this client.

Which is an appropriate nursing action when caring for a client taking benazepril for hypertension? Assess for dizziness. Assess for dark, tarry stools. Administer the medication after meals. Monitor the electroencephalogram (EEG).

Assess for dizziness Rationale Dizziness may occur during the first few weeks of therapy until the client adapts physiologically to the medication. Dark, tarry stools are not a side effect of benazepril. Administering the medication after meals is unnecessary; however, if nausea occurs, the medication may be taken with food or at bedtime. The blood pressure should be monitored before and after administration. An EEG is unnecessary. Cardiac monitoring may be instituted because of possible dysrhythmias.

Which B vitamin deficiency will result in Wernicke encephalopathy? B 3 (niacin) B 1 (thiamine) B 2 (riboflavin) B 6 (pyridoxine)

B 1 (thiamine) Rationale Severe deficiency of thiamine will result in Wernicke encephalopathy. Niacin deficiency causes pellagra. Riboflavin deficiency can result in cutaneous, oral, and corneal changes. Pyridoxine deficiency can progress to sideroblastic anemia, neurological disturbances, and xanthurenic aciduria, among other problems.

The nurse is discharging a client on oral potassium replacement. Which of the following statements requires further teaching by the nurse? A"I can still take my nonsteroidal anti-inflammatory medications occasionally for my arthritis pain." B"I will continue to use salt substitutes to flavor my food." C"I will take my furosemide first thing in the morning." D"I will read the food labels for added potassium."

B"I will continue to use salt substitutes to flavor my food." Rationale: Salt substitutes are made using potassium. As the client is taking potassium supplements, they should avoid salt substitutes to prevent hyperkalemia from occurring. NSAIDS can be used occasionally. The furosemide should be taken in the morning. Some low-sodium prepared foods may contain potassium, so reading the labels is important.

Which complication is an adverse effect of cortisone therapy? Hypoglycemia Severe anorexia Anaphylactic shock Behavioral changes

Behavioral changes Rationale Development of mood swings and psychosis is possible during long-term therapy with glucocorticoids because of fluid and electrolyte alterations. Hypoglycemia, severe anorexia, and anaphylactic shock are not responses to long-term glucocorticoid therapy.

The nurse is talking with a client who was admitted with an acute myocardial infarction due to coronary artery disease. The clients asks what the purpose for the prescribed carvedilol is. How should the nurse respond? A"A beta blocker will prevent postural hypotension." B"Most people develop hypertension after a heart attack." C"This drug will decrease the workload on your heart." D"Beta blockers will help to increase your heart rate."

C"This drug will decrease the workload on your heart." Rationale: One action of beta blockers is to decrease systemic vascular resistance by dilation of the arterioles. This is useful for clients with coronary artery disease and will reduce the risk of another MI or a sudden cardiac event. Some of the more commonly prescribed beta blockers include metoprolol and carvedilol (Coreg). The other responses are incorrect.

A client who was prescribed atorvastatin (Lipitor) one month ago calls the triage nurse at the clinic complaining of muscle pain and weakness in his legs. Which statement reflects the correct drug-specific teaching the nurse should provide to this client? A. Increase consumption of potassium-rich foods since low potassium levels can cause muscle spasms. B. Have serum electrolytes checked at the next scheduled appointment to assess hyponatremia, a cause of cramping. C. Make an appointment to see the healthcare provider, because muscle pain may be an indication of a serious side effect. D. Be sure to consume a low-cholesterol diet while taking the drug to enhance the effectiveness of the drug.

C. Make an appointment to see the healthcare provider, because muscle pain may be an indication of a serious side effect. Myopathy, suggested by the leg pain and weakness, is a serious and potentially life-threatening complication of Lipitor, and should be evaluated immediately by the healthcare provider.

A client is prescribed heparin therapy for a deep vein thrombosis (DVT). Which laboratory value should the nurse monitor closely? AD-dimer BPlatelet count CActivated partial thromboplastin time DBleeding time

CActivated partial thromboplastin time Rationale: Heparin is used to prevent further clots from being formed and to prevent the present clot from enlarging. The activated partial thromboplastin time (APTT) test measures the time it takes blood to clot and is used to monitor the effectiveness of heparin therapy. The therapeutic range is about 1 1/2 to 2 or 2 1/2 times the normal values. D-dimer is used to evaluate blood clot formation. Platelet counts are used to evaluate abnormal bleeding times. Bleeding time refers to the time it takes for a pinprick to stop bleeding (normally, about 2 1/2 minutes).

A client with a history of asthma is admitted for a minor surgical procedure. Preoperatively, the peak flow is measured at 480 liters/minute. Postoperatively, the client reports chest tightness and the peak flow is now 200 liters/minute. What should the nurse do first? ANotify both the surgeon and primary care provider BRepeat the peak flow reading in 30 minutes CAdminister the PRN dose of albuterol DApply oxygen at two liters per nasal cannula

CAdminister the PRN dose of albuterol Rationale: Peak flow monitoring during exacerbations of asthma is recommended for clients with moderate-to-severe persistent asthma to determine the severity of the exacerbation and to guide the treatment. A peak flow reading of less than 50% of the client's baseline reading is a medical alert condition and a short-acting beta agonist must be taken immediately. Notifying the health care provider is important, but that is not what would be done first. First, the client needs assistance. Oxygen administration will not be effective if the airway constriction is not relieved with the albuterol. Leaving the client and returning in 30 minutes will do nothing to help a client in acute distress.

The nurse is caring for a child diagnosed with seizures. While teaching the family and the child about the medication phenytoin, which concept should the nurse emphasize? A Omit the medication if the child is seizure-free B Serve a diet that is high in iron C A rash is normal with this medication D Maintain good oral hygiene and dental care

D Maintain good oral hygiene and dental care Question Explanation Rationale: Gingival hyperplasia may occur with this medication. It is important that good oral hygiene is maintained. The medication should never be stopped, even if the child is seizure-free. A sudden discontinuation could result in status epilepticus. A diet high in iron interferes with phenytoin absorption and will reduce the effectiveness. A blister-like rash is not normal with this medication and could indicate medication-related Stevens-Johnson syndrome, which is a serious disorder of the skin and mucous membranes.

The nurse is preparing a client with rheumatoid arthritis (RA) for discharge to an assisted living facility. Which statement about the prescribed oral glucocorticoid is correct? A"The medication will reverse the joint deterioration of RA." B"You will be taking the medication for several years." C"It is normal to experience some memory loss or hallucinations." D"The medication will be gradually tapered off over 5 to 7 days."

D"The medication will be gradually tapered off over 5 to 7 days." Rationale: RA is an autoimmune, inflammatory disease that affects the joints. It is a progressive disease that causes joint deterioration and destruction, joint deformities and functional limitations for affected clients. The main goal of pharmacotherapy for RA is symptom relief. Glucocorticoids are anti-inflammatory drugs, which can relieve symptoms of RA and may also delay disease progression. For generalized symptoms related to RA, oral glucocorticoids are indicated. The most commonly employed oral glucocorticoids are prednisone and prednisolone. Glucocorticoids can slow disease progression, but will not reverse it. Treatment with glucocorticoids for RA is usually limited to short courses. Adverse psychological reactions such as hallucinations, memory loss or other psychoses must be reported to the provider and may require discontinuation of the glucocorticoid. To minimize adrenal insufficiency when glucocorticoids are discontinued, doses should be tapered very gradually.

The nurse admits a client with tumor-induced spinal cord compression. Which medication should the nurse anticipate to be prescribed to offer the best palliative treatment for this client? A. Morphine sulfate. B. Ibuprofen. C. Amitriptyline. D. Dexamethasone.

D. Dexamethasone. Dexamethasone is a palliative treatment modality to manage symptoms related to compression due to tumor growth. Morphine sulphate is an opioid analgesic used in oncology to manage severe or intractable pain. Ibuprofen, a nonsteroidal antiinflammatory drug (NSAID), provides relief for mild to moderate pain, suppression of inflammation, and reduction of fever. Amitriptyline, a tricyclic antidepressant, is often prescribed for pain related to neuropathic origin and provides a reduction in opioid dosage.

The nurse is caring for an 83-year-old client who is experiencing a sudden onset of confusion. Which medication most likely contributed to this change? ACardiac glycoside BAnticoagulant CLiquid antacid DAntihistamine

DAntihistamine Rationale: Older adults are more susceptible to the side effects of anticholinergic medications, such as antihistamines. Antihistamines often cause confusion in the older adult, especially at high doses. Cardiac glycosides, anticoagulants and antacids are not associated with confusion or mental status changes in the older adult.

The nurse is preparing to administer digoxin to a client admitted for acute decompensated heart failure. Which action is the priority before giving this drug? AMonitor oxygen saturation on room air BAssess the client's weight and compare to the baseline CAuscultate the lungs for crackles in the bases DAssess the apical pulse for a full minute

DAssess the apical pulse for a full minute Rationale: Digoxin, a cardiac glycoside, is used to slow the heart rate and increase the force of contraction. The priority for the nurse is to count the client's apical pulse for one full minute, even if the heart rhythm is regular. Typically, when the pulse is less than 60, digoxin should not be given. The other actions are also appropriate assessments for a client with heart failure. However, they are not the priority when administering digoxin.

A client who is a smoker expresses a desire to postpone her first pregnancy for at least 5 years and declines to use a barrier method. Which method would the nurse anticipate providing education for? A birth control patch A vaginal ring Medroxyprogesterone Combined oral contraceptive pills

Medroxyprogesterone Rationale Medroxyprogesterone is a long-acting progestin-only contraceptive that is less likely to cause cardiovascular problems in women who smoke than contraceptives containing estrogen might. Vaginal rings, combined oral contraceptive pills, and the birth control patch all contain estrogen and are not recommended for women who smoke.

Which sign of hypokalemia will the nurse monitor for in a client receiving furosemide? Chvostek sign Muscle weakness Anxious behavior Abdominal cramping

Muscle weakness Rationale With hypokalemia, failure occurs in myoneural conduction and smooth muscle functioning, resulting in fatigue and muscle weakness. Chvostek sign, the contraction of the facial muscles in response to a light tap over the facial nerve in front of the ear, is associated with hypocalcemia; low calcium levels allow sodium to move into excitable cells, increasing depolarization and nerve excitability. Anxiety and irritability are associated with hyperkalemia. Hyperkalemia affects the nervous and muscular systems; fatigue, weakness, and lethargy are associated with hypokalemia. Decreased gastrointestinal motility occurs with hypokalemia; abdominal cramping is associated with hyperkalemia and is caused by hyperactivity of smooth muscles.

Which body function maintained by thiamine (vitamin B 1) and niacin (vitamin B 3) will the nurse monitor when prescribed for a client with alcoholism? Neuronal activity Bowel elimination Efficient circulation Prothrombin development

Neuronal activity Rationale Thiamine and niacin help convert glucose for energy and influence nerve activity. These vitamins do not affect elimination. These vitamins are not related to circulatory activity. Vitamin K, not thiamine and niacin, is essential for the manufacturing of prothrombin.

Enoxaparin 40 mg subcutaneously daily is prescribed for a client who had abdominal surgery. The nurse explains that the medication is given for which purpose? To control postoperative fever To provide a constant source of mild analgesia To limit the postsurgical inflammatory response To provide prophylaxis against postoperative thrombus formation

To provide prophylaxis against postoperative thrombus formation Rationale Enoxaparin, a low-molecular-weight heparin, prevents the conversion of fibrinogen to fibrin and of prothrombin to thrombin by enhancing the inhibitory effects of antithrombin III. Enoxaparin is not an antipyretic. Enoxaparin is not an analgesic. Enoxaparin is not an anti-inflammatory medication.

Which teaching would a nurse give to a client with a prescription for potassium supplements? To report any abdominal distress To use salt substitutes to season food To take the medication on an empty stomach To increase the dosage if muscle cramps occur

To report any abdominal distress Rationale Potassium supplements can cause gastrointestinal ulceration and bleeding. Most salt substitutes contain potassium, and their use with potassium supplements can cause hyperkalemia. Because they can be irritating to the stomach, potassium supplements should not be taken on an empty stomach. Although muscle cramps may indicate hypokalemia, clients should not adjust their own dosage.

Which method would the nurse recommend when teaching the client with asthma how to determine if an inhaler is empty? Track the number of doses taken. Taste the medication when sprayed into the air. Shake the canister. Place the canister in water to see if it floats.

Track the number of doses taken. Rationale The only way to determine if the canister is empty is to track the number of doses taken. It is wasteful to spray medication into the air; tasting it from the air is not an effective method of determining if the canister is empty. Shaking the canister is not effective; even if there is no more medication, some propellant may be left. It is futile to place the canister in water; the flotation test is ineffective.tory center in the brain.

A 20-year-old woman visiting the clinic says that she wishes to begin using depot medroxyprogesterone acetate as a form of birth control. Which important information would the nurse include when teaching the client about this medication? -'Medroxyprogesterone offers protection against the herpes simplex virus.' -'You will need a repeat injection every 6 months.' -'Increase your intake of iron-rich foods to prevent anemia from increased blood loss during menstruation.' -'Increase your calcium intake and exercise because loss of bone mineral density may occur.'

'Increase your calcium intake and exercise because loss of bone mineral density may occur.' Rationale Loss of bone mineral density is a significant side effect of depot medroxyprogesterone acetate, and increased calcium intake and exercise should be encouraged. Medroxyprogesterone should be administered every 11 to 13 weeks; 6 months is too long before the next dose. Menstrual periods usually lighten or disappear over time. Medroxyprogesterone confers no protection against herpes simplex virus.

The nurse is caring for a 6-year-old child who has undergone craniotomy. The parents ask what effect mannitol has. Which response by the nurse is most appropriate? 'It relieves cerebral pressure.' 'It increases the bladder's filtration rate.' 'It reduces glucose excretion in the urine.' 'It decreases the peripheral retention of fluid.'

'It relieves cerebral pressure.' Rationale Mannitol is an osmotic diuretic used to relieve cerebral edema. The bladder is a storage basin and is not involved with filtration; mannitol acts in the kidneys. Mannitol is an osmotic diuretic that affects neither the body's excretion of glucose nor peripheral edema.

A client asks the nurse what she should do if she forgets to take her contraceptive pill 1 day. Which response by the nurse is appropriate? 'Take your pills as instructed.' 'Call your primary health care provider immediately.' 'Continue as usual, and there shouldn't be a problem.' 'On the next day take 1 pill in the morning and 1 pill before bedtime.'

'On the next day take 1 pill in the morning and 1 pill before bedtime.' Rationale The client should make up for the missed pill by taking 2 pills the next day; taking 1 pill in the morning and 1 pill in the evening decreases the chance of the client becoming nauseated. Telling the client to take her pills as instructed does not explain what is to be done if a pill is missed; missing 1 pill can alter hormone levels and predispose the client to becoming pregnant. It is unnecessary to call the primary health care provider unless other problems are identified. Telling the client that there should be no problem if she continues as usual is incorrect advice; again, missing 1 pill can alter hormone levels and predispose a woman to pregnancy.

390) Which information would the nurse include when preparing a teaching plan for a client prescribed sublingual nitroglycerin? -"Place the tablet under the tongue or between the cheek and gums." -"It takes 30 to 45 minutes for the nitroglycerin to achieve its effect." -"If dizziness occurs, take a few deep breaths and lean the head back." -"To facilitate absorption, drink a large glass of water after taking the medication."

*"Place the tablet under the tongue or between the cheek and gums." Rationale Nitroglycerin sublingual tablets should not be chewed, crushed, or swallowed. They work much faster when absorbed through the lining of the mouth. Clients are instructed to place the tablet under the tongue or between the cheek and gums and let it dissolve. The client should not eat, drink, smoke, or use chewing tobacco while a tablet is dissolving; this will decrease the effectiveness of the medication. Nitroglycerin sublingual tablets usually give relief in 1 to 5 minutes. If a client experiences dizziness or lightheadedness, the client is instructed to take several deep breaths and bend forward with the head between the knees. This position promotes blood flow to the head. If taken with water, the tablet is washed away from the site of absorption or may be swallowed.

A client has been taking rosuvastatin for six weeks as part of a treatment plan to reduce hyperlipidemia. The clinic nurse is reviewing and reinforcing information about the medication with the client. Which statements by the client indicates an understanding about the medication? Select all that apply. -"I will need to call my doctor if I have any muscle weakness or pain, especially in my legs." -"I will need to come back to have my liver and kidney labs checked." -"I need to be careful when I get up because this medication can make my blood pressure drop." -"I add some nuts and fresh fruit to my oatmeal in the morning and I can't remember when I last ate a steak." -"This medication has to be taken first thing in the morning, before I eat breakfast."

-"I will need to call my doctor if I have any muscle weakness or pain, especially in my legs." -"I will need to come back to have my liver and kidney labs checked." -"I add some nuts and fresh fruit to my oatmeal in the morning and I can't remember when I last ate a steak." Rationale: Clients taking rosuvastatin need to be monitored for alteration in liver function. An adverse effect of rosuvastatin is muscle pain and weakness (rhabdomyolysis). Left untreated, rhabdomyolysis can lead to renal impairment. The medication does not affect blood pressure or cause orthostatic hypotension. The client should be taught to follow a low-cholesterol diet, which includes increasing intake of whole grains and limiting intake of foods high in saturated fats, trans fats and dietary cholesterol. The medication is ordered once a day. The client can take it at any time of day, preferably at the same time of day each day, before or after eating.

256) Which information would the nurse provide when administering the first dose of prednisone prescribed to a client with an exacerbation of colitis? -"Prednisone protects you from getting an infection." -"The medication may cause weight loss by decreasing your appetite." -"Prednisone is not curative but does cause a suppression of the inflammatory process." -"The medication is relatively slow in precipitating a response but is effective in reducing symptoms."

-"Prednisone is not curative but does cause a suppression of the inflammatory process." Rationale Prednisone inhibits phagocytosis and suppresses other clinical phenomena of inflammation; this is a symptomatic treatment that is not curative. Prednisone suppresses the immune response, which increases the potential for infection. The appetite is increased with prednisone; weight gain may result from the increased appetite or from fluid retention. Generally, the response to prednisone is rapid.

Which instructions about the use of nitroglycerin to prevent angina will the nurse provide to a client? -'At the point when pain first occurs, place two tablets under the tongue.' -'Place one tablet under the tongue before activity, and swallow another if pain occurs.' -'Before physical activity, place one tablet under the tongue, and repeat the dose in 5 minutes if pain occurs.' -'Place one tablet under the tongue when pain occurs and use an additional tablet after the attack to prevent recurrence.'

-'Before physical activity, place one tablet under the tongue, and repeat the dose in 5 minutes if pain occurs.' Rationale Anginal pain, which can be anticipated during certain activities, may be prevented by dilating the coronary arteries immediately before engaging in the activity. Generally, one tablet is administered at a time; doubling the dosage may produce severe hypotension and headache. The sublingual form of nitroglycerin is absorbed directly through the mucous membranes and should not be swallowed. When the pain is relieved, rest generally will prevent its recurrence by reducing oxygen consumption of the myocardium.

Which instruction would the nurse include when teaching the client about sublingual nitroglycerin? -'Once the tablet is dissolved, spit out the saliva.' -'Take tablets 3 minutes apart up to a maximum of five tablets.' -'Common side effects include headache and low blood pressure.' -Once opened, the tablets should be refrigerated to prevent deterioration.'

-'Common side effects include headache and low blood pressure.' Rationale The primary side effects of nitroglycerin are headache and hypotension. It is not necessary to spit out saliva into which nitroglycerin has dissolved. For pain that is not relieved, additional tablets may be taken every 5 minutes up to a total of three tablets. It should be stored at room temperature.

The nurse provides discharge medication education to a client who has a prescription for warfarin. Which client statement indicates to the nurse that teaching was effective? -'I will avoid taking aspirin and nonsteroidal anti-inflammatory drugs [NSAIDs].' -'I will need to develop a more sedentary routine.' -'I will need to have regular complete blood counts to guide warfarin dosage.' -'Before going to the dentist, I will ask my health care provider for antibiotics.'

-'I will avoid taking aspirin and nonsteroidal anti-inflammatory drugs [NSAIDs].' Rationale Acetaminophen should be used when an analgesic is required because it does not interfere with platelet aggregation. Acetylsalicylic acid (aspirin) should be avoided because it interferes with platelet aggregation. Immobility causes venous pooling and can predispose the client to deep vein thrombosis. Antibiotics are not necessary when going to the dentist; this is done when clients have cardiac problems, such as rheumatic fever or cardiac surgery. A prothrombin time (PT) or international normalized ratio (INR), not a complete blood count, needs to be done periodically.

Potassium supplements are prescribed for a client receiving diuretic therapy. Which client statement indicates that the teaching about potassium supplements is understood? -'I will report any abdominal distress.' -'I should use salt substitutes with my food.' -'The medication must be taken on an empty stomach.' -'The dosage is correct if my urine output increases.'

-'I will report any abdominal distress.' Rationale Potassium supplements can cause gastrointestinal ulceration and bleeding. Most salt substitutes contain potassium, and their use with potassium supplements can cause hyperkalemia. Because they can be irritating to the stomach, potassium supplements should not be taken on an empty stomach. An increase in urine output is the therapeutic effect of diuretic therapy, not potassium supplements. An adverse effect of potassium supplements is oliguria.

The health care provider prescribes isosorbide dinitrate 10 mg for a client with chronic angina pectoris. The client asks the nurse why the isosorbide dinitrate is prescribed. How will the nurse respond? -'It prevents excessive blood clotting.' '-It suppresses irritability in the ventricles.' -'It decreases cardiac oxygen demand.' -'The inotropic action increases the force of contraction of the heart.'

-'It decreases cardiac oxygen demand.' Rationale Isosorbide dinitrate dilates peripheral veins and arteries thus decreasing preload and decreasing oxygen demand. Preventing blood from clotting is the action of anticoagulants. Suppressing irritability in the ventricles is the action of antidysrhythmics. Increasing the force of contraction of the heart is the action of cardiac glycosides.

The nurse is preparing a teaching plan for a client prescribed nitroglycerin sublingual. Which would the nurse include in the teaching? -'Place the tablet under the tongue or between the cheek and gums.' -'It takes 30 to 45 minutes for the nitroglycerin to achieve its effect.' -'If dizziness occurs, take a few deep breaths and lean the head back.' -'To facilitate absorption, drink a large glass of water after taking the medication.'

-'Place the tablet under the tongue or between the cheek and gums.' Rationale Nitroglycerin sublingual tablets should not be chewed, crushed, or swallowed. They work much faster when absorbed through the lining of the mouth. Clients are instructed to place the tablet under the tongue or between the cheek and gums and let it dissolve. The client should not eat, drink, smoke, or use chewing tobacco while a tablet is dissolving; this will decrease the effectiveness of the medication. If taken with water, the tablet is washed away from the site of absorption or may be swallowed. Nitroglycerin sublingual tablets usually give relief in 1 to 5 minutes. If a client experiences dizziness or lightheadedness, the client is instructed to take several deep breaths and bend forward with the head between the knees. This position promotes blood flow to the head.

1078) A client with hypertension is prescribed an angiotensin II receptor blocker (ARB). Which instructions will the nurse provide about this medication? Select all that apply. One, some, or all responses may be correct. -'Monitor the blood pressure daily.' -'Stop treatment if a cough develops.' -'Stop the medication if swelling of the mouth, lips, or face develops.' -'Have blood drawn for potassium levels 2 weeks after starting the medication.' -'Do not take nonsteroidal anti-inflammatory drugs (NSAIDs) concurrently with this medication.'

-'Stop the medication if swelling of the mouth, lips, or face develops.' -'Have blood drawn for potassium levels 2 weeks after starting the medication.' Rationale The medication should be stopped if angioedema occurs, and the health care provider should be notified. Electrolyte levels of potassium, sodium, and chloride should be obtained 2 weeks after the start of therapy and then periodically thereafter. Daily monitoring of blood pressure is not indicated. There is no need to avoid the use of NSAIDs while taking an ARB. A dry cough may occur during treatment with ARBs; however, it is not necessary to discontinue the medication because the cough usually resolves.

A female client receiving cortisone therapy for adrenal insufficiency expresses concern that she is developing facial hair. How would the nurse respond? -'It is just another sign of adrenal insufficiency.' -'This side effect will disappear after therapy.' -'This is not important as long as you are feeling better.' -'The medication contains a hormone that causes male characteristics.'

-'The medication contains a hormone that causes male characteristics.' Rationale Some cortisol derivatives possess 17-keto-steroid (androgenic) properties, which result in hirsutism. Facial hair is not a sign of the illness; it results from androgens that are present in cortisol. Hirsutism will be a long-term problem because therapy is provided on a long-term, usually lifelong, basis. The response 'This is not important as long as you are feeling better' doesn't address the client's concerns

A client comes in for a pregnancy test. She tells the nurse that pregnancy may have occurred because she missed her contraceptive pills for 1 week when she had the flu. Which response by the nurse is appropriate?' ='That's the trouble with using contraceptive pills. People frequently forget to take them.' -'You may be correct. The effect of contraceptive pills depends on them being taken on a regular schedule.' -'Let's find out whether you really are pregnant. -If you are, you may want to consider having an abortion.' '-Contraceptive pills are unpredictable. You could have become pregnant even if you had taken them regularly.'

-'You may be correct. The effect of contraceptive pills depends on them being taken on a regular schedule.' Rationale An oral contraceptive program requires the client to take one tablet daily from the fifth day of the cycle and continue taking tablets for 20 or 21 days. Interrupting the monthly dosage program may permit release of luteinizing hormone, resulting in ovulation and possibly pregnancy. Stating that people often forget to take oral contraceptive pills is judgmental; contraceptive practice is the client's choice. It is premature to discuss abortion. Oral contraceptives that are taken on an exact schedule have a very high rate of success.

Which explanation would the nurse provide for administering prednisone to a client with an exacerbation of colitis? -The client will be protected from getting an infection. -Symptoms associated with the colitis will decrease slowly over time. -Although the medication causes anorexia, weight loss may not occur. -Although the medication decreases intestinal inflammation, it will not cure the colitis.

-Although the medication decreases intestinal inflammation, it will not cure the colitis. Rationale Prednisone inhibits phagocytosis and suppresses other clinical phenomena of inflammation; this is a symptomatic treatment that is not curative. The medication suppresses the immune response and increases the potential for infection. The response usually is rapid. Appetite is increased; weight gain may result from this or from fluid retention.

The nurse is teaching a client with stable angina about their new prescription for nitroglycerin transdermal patch. Which instructions should the nurse include? Select all that apply. -Remove the patch if ankle edema occurs -Apply the patch to a hairless area of the body -Notify your provider for persistent dizziness or any fainting episode -Apply a second patch with chest pain -Plan for patch-free time, usually overnight -Rotate the application area

-Apply the patch to a hairless area of the body -Notify your provider for persistent dizziness or any fainting episode -Plan for patch-free time, usually overnight -Rotate the application area Rationale: Nitroglycerin (NTG) acts directly on vascular smooth muscle to promote vasodilation. It decreases the pain of exertional angina primarily by decreasing cardiac oxygen demand. NTG comes in a variety of routes of administration. NTG patches contain a reservoir from which the drug is slowly released. Following release, the drug is absorbed through the skin and then into the blood. The rate of release is constant and, depending on the patch used, can range from 0.1 to 0.8 mg/ hr. Effects begin within 30 to 60 minutes and persist as long as the patch remains in place (up to 14 hours). Patches are applied once daily to a hairless area of skin. The site should be rotated to avoid local irritation. Tolerance develops if patches are used continuously (24 hours a day every day). Accordingly, a daily "patch-free" interval of 10 to 12 hours is recommended. This can be accomplished by applying a new patch each morning, leaving it in place for 12 to 14 hours, and then removing it in the evening. NTG can cause orthostatic hypotension and the client should let their provider know if dizziness and lightheadedness persist or the client has a fainting (syncopal) episode as these may indicate that the NTG dose needs to be adjusted/decreased. The other instructions are not appropriate for this medication.

A client with an intravenous (IV) infusion containing 40 mEq of potassium reports a stinging pain at the IV site. Which actions will the nurse take? Select all that apply. One, some, or all responses may be correct. -Restart the IV in a different vein. -Assist the client through guided imagery. -Assess the IV site. -Ask the health care provider for pain medication. -Verify that the potassium is adequately diluted and not infusing too rapidly.

-Assess the IV site. -Verify that the potassium is adequately diluted and not infusing too rapidly. Rationale It is important to first make sure that the IV catheter is patent and that there is no infiltration. The potassium dosage is large and can be very irritating to veins if it isn't sufficient diluted or if it infuses too rapidly. A 40-mEq dose should be diluted in at least 1 L of IV solution. Although imagery may help distract the client from discomfort, this response provides no information as to why the stinging sensation is occurring. Asking the provider for an analgesic doesn't address the underlying problem.

The nurse is preparing to discharge a client who presented to the emergency room for an acute asthma attack. The nurse notes that upon discharge the health care provider has prescribed theophylline 300 mg orally to be taken daily at 9:00 AM. The nurse will teach the client to take the medication on which schedule? -One hour before or 2 hours after eating -At bedtime -At the specific time prescribed -Daily until symptoms are gone

-At bedtime Rationale For theophylline to be effective, therapeutic serum levels must be maintained by taking the medication at the prescribed time. If the medication is not taken at the prescribed time, the level may drop below the therapeutic range. The medication will not be effective if it drops below the therapeutic range. Theophylline should be given after a meal and with a full glass of water to decrease gastric irritability. Giving it 2 hours after a meal (on an empty stomach) can result in gastric discomfort. It should not be taken at night, because it can cause central nervous system stimulation resulting in insomnia, restlessness, irritability, etc. Theophylline is used for long-term medication therapy.

Which instructions will the nurse include in the teaching plan for a client who will be taking simvastatin? Select all that apply. One, some, or all responses may be correct. -Increase dietary intake of potassium. -Avoid prolonged exposure to the sun. -Schedule regular ophthalmic examinations. -Take the medication at least half an hour before meals. -Contact your health care provider if skin becomes gray-bronze.

-Avoid prolonged exposure to the sun. -Schedule regular ophthalmic examinations. -Contact your health care provider if skin becomes gray-bronze. Rationale Simvastatin increases photosensitivity; the client should avoid sun exposure and use sunblock. The client should be monitored for the adverse effects of glaucoma and cataracts. Gray-bronze skin and unexplained muscle pain are signs of rhabdomyolysis. Rhabdomyolysis, a life-threatening response, is the disintegration of muscle associated with myoglobin in the urine. Simvastatin does not affect levels of potassium. The medication is most effective when taken at bedtime because cholesterol synthesis is highest at night.

The nurse is administering an osmotic diuretic to a client with a traumatic brain injury. Which finding best indicates that the medication was effective? -A 250 mL clear, yellow urine output over four hours -B Clear bilateral lung sounds to posterior auscultation -D Bilateral ovoid pupils that are slow to constrict

-C Intracranial pressure reading of 14 mmHg Rationale: Osmotic diuretics, such as mannitol, are used to reduce intracranial or intraocular pressure. Intracranial pressure (ICP) for a client with a head injury should be less than 20 mmHg and the osmotic diuretic may be administered to reduce a high ICP. The osmotic diuretic will reduce the amount of water normally reabsorbed by the renal tubules and loop of Henle, so urinary output is increased, which is an expected occurrence, but does not indicate effectiveness of the medication. Ovoid pupils may indicate the presence of cerebral hypertension. An osmotic diuretic is not intended to reduce pulmonary edema, thus clear lung sounds are not an indicator for effectiveness of the diuretic for this particular client.

Atenolol is prescribed for a client with moderate hypertension. Which information would the nurse include when teaching the client about this medication? Select all that apply. One, some, or all responses may be correct. -Change to standing positions slowly. -Take the medication before going to bed. -Count the pulse before taking the medication. -Mild weakness and fatigue are common side effects. -It is safe to take over-the-counter (OTC) medications.

-Change to standing positions slowly. -Count the pulse before taking the medication. -Mild weakness and fatigue are common side effects. Rationale A side effect of this medication is orthostatic hypotension. The client should be advised to move to a standing position slowly to allow the body to adjust to the new position. The rate of the pulse should be taken before administering the medication; slower rates due to ventricular dysrhythmias and heart block may occur. Mild weakness and fatigue are side effects of this medication. The blood pressure decreases when the client is sleeping; the medication usually is prescribed to be administered earlier in the day. The medication should be taken with food. No OTC medication should be taken without consulting the prescribing health care provider; decreased or increased effects can occur when there is an interaction with another medication.

The client with congestive heart failure is receiving furosemide 80 mg once daily. Which data collection assessment would be performed to evaluate medication effectiveness? Select all that apply. One, some, or all responses may be correct. -Daily weight -Intake and output -Monitor for edema -Daily pulse oximetry -Auscultate breath sounds

-Daily weight -Intake and output -Monitor for edema -Daily pulse oximetry -Auscultate breath sounds Rationale Daily weight at the same time, on the same scale, and in the same clothing is important as it is an indication of fluid gains or losses. The nurse would also record daily intake and output and report intake exceeding output. The nurse would monitor for peripheral edema and document the findings. It is important to obtain and record vital signs and daily pulse oximetry as improving results relate to effectiveness of furosemide. The nurse would also auscultate breath sounds, look for jugular venous distension, and report abnormal data.

A client who had a myocardial infarction receives a prescription for a nitroglycerin patch. Which statement would the nurse identify as the purpose of the nitroglycerin patch? -Decreased heart rate lowers cardiac output. -Increased cardiac output increases oxygen demand. -Decreased cardiac preload reduces cardiac workload. -Peripheral venous and arterial constriction increases peripheral resistance.

-Decreased cardiac preload reduces cardiac workload. Rationale Nitroglycerin reduces cardiac workload by decreasing the preload of the heart by its vasodilating effect. It decreases blood pressure, not heart rate (which may increase to compensate for the decreased blood pressure). It decreases, not increases, oxygen demand. Nitroglycerin dilates, not constricts, peripheral veins and arteries.

Which goal is the priority for a client with asthma who has a prescription for an inhaled bronchodilator? -Is able to obtain pulse oximeter readings -Demonstrates use of a metered-dose inhaler -Knows the health care provider's office hours -Can identify triggers that may cause wheezing

-Demonstrates use of a metered-dose inhaler Rationale Clients with asthma use metered-dose inhalers to administer medications prophylactically or during times of an asthma attack; this is an important skill to have. Home management typically includes self-monitoring of the peak expiratory flow rate rather than pulse oximetry. Although knowing the health care provider's office hours is important, it is not the priority; during a persistent asthma attack that does not respond to planned interventions, the client should go to the emergency department of the local hospital or call 911 for assistance. Although it is important to be able to identify triggers that may cause wheezing, knowing these cannot prevent all wheezing; therefore, being able to abort wheezing with a bronchodilator is the greater priority.

A client is receiving metoprolol. Which potential effect will the nurse teach the client to expect? -Dizziness with strenuous activity -Acceleration of the heart rate after eating a heavy meal -Flushing sensations after taking the medication -Pounding of the heart

-Dizziness with strenuous activity Rationale Because metoprolol competes with catecholamines at beta-adrenergic receptor sites, the expected increase in the heart's rate and contractility in response to exercise does not occur. This, combined with the medication's hypotensive effect, may lead to dizziness. Metoprolol decreases the heart rate. Flushing sensations and pounding of the heart do not represent side effects of metoprolol.

A client with hypertension has received a prescription for metoprolol. Which information will the nurse include when teaching this client about metoprolol? -Do not abruptly discontinue the medication. -Consume alcoholic beverages in moderation. -Report a heart rate of less than 70 beats per minute. -Increase the medication dosage if chest pain occurs.

-Do not abruptly discontinue the medication. Rationale Abrupt discontinuation of metoprolol may cause rebound hypertension and an acute myocardial infarction. Alcohol is contraindicated for clients taking beta-adrenergic blockers such as metoprolol. Clients should never increase medications without medical direction. The pulse rate can go lower than 70 beats per minute as long as the client is asymptomatic.

A health care provider prescribes mannitol for a client with a head injury. Which mechanism of action is responsible for therapeutic effects of this medication? -Decreasing the production of cerebrospinal fluid -Limiting the metabolic requirements of the brain -Drawing fluid from brain cells into the bloodstream -Preventing uncontrolled electrical discharges in the brain

-Drawing fluid from brain cells into the bloodstream Rationale Mannitol, an osmotic diuretic, pulls fluid from the brain to relieve cerebral edema. Mannitol's diuretic action does not decrease the production of cerebrospinal fluid. Mannitol does not affect brain metabolism; rest and lowered body temperature reduce brain metabolism. Preventing uncontrolled electrical discharges in the brain is the action of phenytoin sodium, not mannitol.

Which intervention would the nurse implement for a client admitted for an exacerbation of asthma? -Determine the client's emotional state. -Give prescribed medications to promote bronchiolar dilation. -Provide education about the effect of a family history. -Encourage the client to use an incentive spirometer routinely.

-Give prescribed medications to promote bronchiolar dilation. Rationale Asthma involves spasms of the bronchi and bronchioles as well as increased production of mucus; this decreases the size of the lumina, interfering with inhalation and exhalation. Bronchiolar dilation will reduce airway resistance and improve the client's breathing. Although identifying and addressing a client's emotional state is important, maintaining airway and breathing are the priority. In addition, emotional stress is only one of many precipitating factors, such as allergens, temperature changes, odors, and chemicals. Although recent studies indicate a genetic correlation along with other factors that may predispose a person to the development of asthma, exploring this issue is not the priority. The use of an incentive spirometer is not helpful because of mucosal edema, bronchoconstriction, and secretions, all of which cause airway obstruction.

Which action would be the most appropriate way for the nurse to evaluate a child's understanding of how to use an inhaler? -Asking questions about using the inhaler -Having the child demonstrate inhaler use -Explaining how the inhaler will be used at home -Having the child tell the nurse about the technique that was learned

-Having the child demonstrate inhaler use Rationale The nurse can best evaluate teaching by asking the learner for a return demonstration. Behavior, rather than words, more easily shows what has been learned. A child may be too young to know whether he or she has any questions. A demonstration, rather than an explanation, can be evaluated more readily. Telling the nurse about the technique that was learned is difficult for a younger child; the ability to articulate a concept is not that advanced—nor is the vocabulary.

958) Warfarin is prescribed for a client who has been receiving intravenous (IV) heparin for a partial occlusion of the left common carotid artery. The client expresses concern about why both medications are needed at the same time. Which rationale would the nurse include to address the client's concern? -This permits the administration of smaller doses of each medication. -Giving both medications allows clot dissolution while preventing new clot formation. -Heparin provides anticoagulant effects until warfarin reaches therapeutic levels. -Administration of heparin with warfarin provides immediate and maximum protection against clot formation.

-Heparin provides anticoagulant effects until warfarin reaches therapeutic levels. Rationale Warfarin is administered orally for 2 to 3 days to achieve the desired effect on the international normalized ratio (INR) level before heparin is discontinued. These medications do not dissolve clots already present. Because each medication affects a different part of the coagulation mechanism, dosages must be adjusted separately. That this approach immediately provides maximum protection against clot formation is not the reason for the administration of both medications; warfarin will not exert an immediate therapeutic effect.

A health care provider prescribes simvastatin 20 mg daily for elevated cholesterol and triglyceride levels for a female client. Which advice is important for the nurse to teach when the client initially takes the medication? -Take the medication with breakfast. -Have liver function tests every 6 months. -Wear sunscreen to prevent photosensitivity reactions. -Inform the health care provider if you wish to become pregnant.

-Inform the health care provider if you wish to become pregnant. Rationale Simvastatin is a teratogen that is contraindicated in pregnancy because it is capable of causing fetal damage. Simvastatin should be taken in the evening because most cholesterol is synthesized between midnight and 3:00 AM. Liver function tests should be done at 6 to 12 weeks initially and only then every 6 months. Although wearing sunscreen should be taught, sensitivity reactions are a rare occurrence, and this is not as important.

The nurse provides instructions about how to use a metered-dose inhaler (MDI) to a client with chronic obstructive pulmonary disease. The nurse concludes that additional teaching is needed when the client demonstrates which technique? -Places the tip of the inhaler just past the lips -Holds the inspired breath for at least 3 seconds -Activates the inhaler during inspiration -Inhales rapidly with the lips sealed around the nebulizer opening

-Inhales rapidly with the lips sealed around the nebulizer opening Rationale The client should inhale slowly rather than rapidly when using a metered-dose inhaler (MDI) to optimize delivery of the nebulized medication into the lungs. If the client has a dry powder inhaler (DPI), then rapid inhaling would be an important action because the powder is not nebulized. The MDI should be gently held in the mouth just past the lips to deliver the medication into the airway. Holding the inspired breath for at least 3 seconds promotes contact of the medication with the bronchial mucosa. The inhaler should be activated during inspiration.

A client receiving corticosteroid therapy states, 'I have difficulty controlling my temper, which is so unlike me, and I don't know why this is happening.' How will the nurse respond? -Tell the client it is nothing to worry about. -Reassure that everyone does this at times. -Instruct the client to attempt to avoid situations that cause irritation. -Inquire about mood swings.

-Inquire about mood swings. Rationale Corticosteroids increase the excitability of the central nervous system, which can cause labile emotions manifested as euphoria and excitability or depression. Telling the client it is nothing to worry about or that it is normal denies the value of the client's statement and offers false reassurance. The client has already stated the problem and does not know why this is happening. Instructing the client to attempt to avoid situations that cause irritation is impractical because the mood swings may occur without an overt cause.

Intravenous (IV) potassium is prescribed for a client with a diagnosis of hypokalemia. Which statement about administration of IV potassium is accurate? -Oliguria is an indication for withholding IV potassium. -Rapid infusion of potassium prevents burning at the IV site. -Clients with severe deficits should be given IV push potassium. -Average IV dosage of potassium should not exceed 60 mEq in 1 hour.

-Oliguria is an indication for withholding IV potassium. Rationale Potassium chloride should not be given unless renal flow is adequate; otherwise, the potassium chloride will accumulate in the body, causing hyperkalemia. Rapid infusion may cause severe pain at the infusion site and precipitate cardiac arrest. Potassium chloride must be well diluted or it will precipitate cardiac arrest. A dose of 60 mEq per hour of potassium chloride is too high.

Sublingual nitroglycerin has been prescribed for a client with unstable angina. Which client response indicates that nitroglycerin is effective? -Pain subsides as a result of arteriole and venous dilation. -Pulse rate increases because the cardiac output has been stimulated. -Sublingual area tingles because sensory nerves are being triggered. -Capacity for activity improves as a response to increased collateral circulation.

-Pain subsides as a result of arteriole and venous dilation. Rationale Nitroglycerin causes vasodilation, increasing the flow of blood and oxygen to the myocardium and reducing anginal pain. An increased pulse rate does not indicate effectiveness; it is a side effect of nitroglycerin. The tingling indicates that the medication is fresh; relief of pain is the only indicator of effectiveness. Nitroglycerin does not promote the formation of new blood vessels.

Which action will a nurse take when a male client receiving prolonged steroid therapy complains of always being thirsty and urinating frequently? -Have the client assessed for an enlarged prostate. -Obtain a urine specimen from the client to test for ketonuria. -Perform a finger stick to test the client's blood glucose level. -Assess the client's lower extremities for the presence of pitting edema.

-Perform a finger stick to test the client's blood glucose level. Rationale The client has signs of an increased serum glucose level, which may result from steroid therapy; testing the blood glucose level is a method of gathering more data. The symptoms are not those of benign prostatic hyperplasia. The blood glucose level, not the amount of ketones in the urine, should be assessed. The symptoms presented are not those of fluid retention but of hyperglycemia.

Which information would the nurse include when teaching a client about warfarin? -Periodic blood testing is necessary. -Increase intake of green leafy vegetables. -Limit the amount of daily physical activity. -It should be continued for minor surgical procedures.

-Periodic blood testing is necessary. Rationale Testing is essential to determine dosing; a therapeutic prothrombin time (PT) ranges from 1.3 to 1.5 times greater than the control and is equal to an international normalized ratio (INR) of 2 to 3 times control. Green leafy vegetables are high in vitamin K, which may decrease medication effectiveness if eaten in large amounts. Physical activities do not need to be limited; however, the type (e.g., contact sports such as football) may need to be restricted. Warfarin will need to be stopped for most dental, medical, and surgical procedures; the provider should be contacted regarding the need to hold the medication.

A client has a prescription for a sublingual nitroglycerin tablet. Which technique will the nurse teach the client to use? -Place the pill inside the cheek and let it dissolve. -Place the pill under the tongue and let it dissolve. -Chew the pill thoroughly and then swallow it. -Swallow the pill with a full glass of water.

-Place the pill under the tongue and let it dissolve. Rationale Sublingual medication is placed under the tongue and is quickly absorbed through the mucous membranes into blood. The buccal route requires placing medication between the cheek and gums. Chewing the pill and then swallowing it may be done for oral administration of some large pills, but not with the sublingual route of administration. Taking the pill with water is required with the oral route of administration of medication, but not with sublingual. In addition, a full glass of water may be an excessive amount of fluid to swallow one pill.

A health care provider prescribes dexamethasone for a client with head trauma. The nurse recognizes that it reduces swelling in the brain by which process? -Acts as a hyperosmotic diuretic -Increases resistance to infection -Reduces the inflammatory response of tissues -Decreases the formation of cerebrospinal fluid

-Reduces the inflammatory response of tissues Rationale Corticosteroids act to decrease inflammation, which decreases edema. Dexamethasone is an anti-inflammatory agent, not a diuretic. Resistance to infection is decreased, not increased, with a corticosteroid. The client's problem is not with increased cerebrospinal fluid.

890) Which instruction will the nurse include in a teaching plan for a client taking a calcium channel blocker such as nifedipine? Select all that apply. One, some, or all responses may be correct. -Reduce calcium intake. -Report peripheral edema. -Expect temporary hair loss. -Avoid drinking grapefruit juice. -Change to a standing position slowly.

-Report peripheral edema -Avoid drinking grapefruit juice. -Change to a standing position slowly. Rationale Changing positions slowly helps reduce orthostatic hypotension. Peripheral edema may occur as a result of heart failure and must be reported. Grapefruit juice affects the metabolism of calcium channel blockers and should be avoided. Reducing calcium intake is unnecessary because calcium levels are not affected. Hair loss does not occur.

A client who recently started receiving oral corticosteroids for a severe allergic reaction is instructed that the dosage will be reduced gradually until all medication is stopped at the end of 2 weeks. Which reason would the nurse provide for this gradual reduction in dosage? -Discontinuing the medication too fast will cause the allergic reaction to reappear. -Slow reduction of the medication will prevent a physiological crisis because the adrenal glands are suppressed. -The health care provider is attempting to determine the minimal dose that will be effective for the allergy. -Sudden cessation of the medication will cause development of serious side effects, such as moon face and fluid retention.

-Slow reduction of the medication will prevent a physiological crisis because the adrenal glands are suppressed. Rationale The body's natural corticosteroid production has been suppressed during treatment; avoiding abrupt cessation of the medication will give the body time to adjust to less and less of the exogenous source and to resume secretion of endogenous corticosteroid. Not completing the course of therapy, rather than stopping it quickly, may cause signs and symptoms of the allergy to recur. The health care provider has already determined the correct dosage, and it has been prescribed. Moon face and fluid retention are associated with long-term steroid use, not with the cessation of therapy.

A client who is on long-term corticosteroid therapy after an adrenalectomy is admitted to the surgical intensive care unit after being involved in a motor vehicle crash. Which statement is an important concern for client safety? -The dosage of steroids will have to be tapered down slowly to prevent acute adrenal crisis. -Steroid therapy will need to be increased to avert a life-threatening crisis. -Osteoporosis secondary to long-term corticosteroids increases fracture risk. -The client is at risk for Cushing syndrome if taking long-term corticosteroid therapy.

-Steroid therapy will need to be increased to avert a life-threatening crisis. Rationale Clients with adrenocorticoid insufficiency who are receiving steroid therapy require increased amounts of medication during periods of stress because they are unable to produce the excess needed by the body. With severe stress, a failure to ensure adequate corticosteroid levels can be life-threatening. Increased stress requires an increase, not a decrease, in glucocorticoids. Although osteoporosis may have contributed to fractures secondary to trauma, this does not present a current risk. Cushing syndrome is a problem with excess corticosteroid therapy, but after an adrenalectomy, the corticosteroid is given in amounts sufficient to replace what the body cannot produce.

Which nursing assessment would performed by a nurse before administering intravenous (IV) infusion of potassium chloride (KCl) 40 mEq in 100 mL of 5% dextrose and water to be infused over 2 hours? Select all that apply. One, some, or all responses may be correct. Urinary output Deep tendon reflexes Last bowel movement Arterial blood gas results Last serum potassium level Patency of the intravenous access

-Urinary output -Last serum potassium level -Patency of the intravenous access Rationale Before administering IV potassium, the urinary output must be normal. If the urine output is low, a potassium infusion may damage renal cells. The last serum potassium level should also be checked to ensure potassium replacement is appropriate. A patent IV access is essential because potassium is very irritating and painful to subcutaneous tissue. The infusion of KCL 40 mEq in 100 mL of 5% dextrose and water has no direct effect on bowel movement patterns, arterial blood gases, or deep tendon reflexes. These items are not required to be assessed before the administration of this medication.

Which assessment would be brought to the health care provider's attention before administration of intravenous potassium chloride? -Progressively worsening muscle weakness -Poor tissue turgor with tenting -Urinary output of 200 mL during the previous 8 hours -Oral fluid intake of 300 mL during the previous 12 hours

-Urinary output of 200 mL during the previous 8 hours Rationale Decreased urinary output may result in the retention of potassium, causing hyperkalemia. Progressively worsening muscle weakness is a manifestation of hypokalemia, which is the reason for prescribing the potassium. Reporting poor tissue turgor with tenting is unnecessary; this may indicate dehydration, which is probably the rationale for the fluid prescribed. Reporting an oral fluid intake of 300 mL during the previous 12 hours is unnecessary; this can precipitate dehydration or can compound an existing dehydration, which can be treated with appropriate hydration.

The nurse is preparing to apply nitroglycerin ointment. Before applying the ointment, which action will the nurse take? -Assess the client's pulse rate. -Prepare the site with an alcohol swab. -Shave the client's chest in the area for application. -Use the dose measuring application paper and spread the ointment in a thin layer to the prescribed amount.

-Use the dose measuring application paper and spread the ointment in a thin layer to the prescribed amount. Rationale The nurse would use the dose measuring application paper supplied with the ointment and spread in a thin layer to the prescribed amount and place side down on the desired skin. The nurse would assess blood pressure reading, not pulse rate. There is no need to clean the site with alcohol before administration. Shaving is not recommended; a hairless site on the chest, back, abdomen, or anterior thigh should be selected.

The nurse is educating a client about how to use a metered-dose inhaler with spacer. Place each step in the correct order by entering the numbers in order. Only enter numbers, no spaces or commas. 1. Breathe out slowly 2. Breathe in deeply 3. Release the medication into the spacer 4. Remove the mouthpiece from the lips 5. Hold breath for 10 seconds

3. Release the medication into the spacer 2. Breathe in deeply 4. Remove the mouthpiece from the lips 5. Hold breath for 10 seconds 1. Breathe out slowly Rationale: Release the medication into the spacer. Breathe in deeply. Remove the mouthpiece, then hold breath for 10 seconds, then breathe out slowly. Spacers are highly recommended when inhalers are used because they increase the availability of the medication to the client.

The nurse is providing discharge education to a client diagnosed with coronary artery disease. The client is prescribed to use a nitroglycerin transdermal patch at home. Which statement by the client indicates a correct understanding of safe medication administration? A"I will remove the old patch and cleanse the area before applying a new patch." B"This drug can lead to hypertension. So, I will monitor my blood pressure at home." C"I will keep a record of chest pain occurrences now that I have this patch." D"I can place this patch on broken skin. It will absorb better."

A"I will remove the old patch and cleanse the area before applying a new patch." Rationale: Numerous administration errors have been reported with nitroglycerin paste and patches. The errors include improper storage and basic administration. The client should be taught to remove the previous patch before applying the new patch and to properly label the tube of nitroglycerin paste and keep it out of the reach of children. When selecting an area to place the patch, the skin should be intact and show no signs of irritation. Nitroglycerin paste has been used erroneously as lotion and caused toxic effects. Nitroglycerin causes vasodilation, which increases the blood supply through the coronary arteries. This may cause hypotension in clients. Some other common side effects include lightheadedness, nausea, dizziness, headache and redness or irritation of the skin covered by the patch.

The home health nurse is completing a medication reconciliation of a client who has a new prescription for warfarin. Which medication should the nurse question the healthcare provider about? AAspirin BNifedipine CNPH insulin DVitamin D supplement

AAspirin Rationale: Warfarin is an anticoagulant that prevents blood from clotting by blocking the synthesis of vitamin K. Clients taking warfarin are at increased risk for bleeding. Aspirin, which is an anti-platelet aggregation, prevents platelets from clumping together. Taking warfarin and aspirin together could increase the risk of bleeding and should be questioned. Nifedipine is a calcium channel blocker and does not interact with warfarin. Insulin and vitamin D supplement do not cause adverse effects when taken with warfarin.

A nurse is providing dietary instructions to a client who is taking prescribed amiloride. Which information will the nurse include in the teaching? AAvoid eating foods that are rich in potassium such as bananas BIt is important to control high-sodium foods such as canned soups CEat plenty of foods that contain calcium such as milk DChoose foods that are high in iron content such as shellfish

AAvoid eating foods that are rich in potassium such as bananas Rationale: Amiloride is a potassium-sparing diuretic used in the treatment of edema, hypertension, and potassium loss caused by other diuretic medications. Amiloride may cause hyperkalemia, so the client should be informed to limit their potassium intake. Sodium, calcium, and iron are not affected by the use of amiloride.

During morning rounds, a healthcare provider informs a client with hypertension that a calcium channel blocker will be added to their treatment regimen. The nurse notes a new prescription for amiloride 10 mg PO daily. Which action does the nurse perform next? AClarify the prescription with the healthcare provider BEducate the client on the new prescription CAdminister the medication with food DAssess the client blood pressure

AClarify the prescription with the healthcare provider Rationale: The nurse should clarify the new prescription. Amiloride is a potassium-sparing diuretic. It is also a look-alike/sound-alike medication commonly confused with amlodipine, a calcium channel blocker. Educating the client on a new prescription, administering the medication with food, and assessing the blood pressure are important interventions for amiloride. However, this prescription should be clarified.

The nurse is monitoring an older adult client prescribed diphenhydramine for contact dermatitis related to poison ivy exposure. Which finding should be reported to the provider as a potential drug-related side effect? AConfusion BHypertension CIncontinence DBradypnea

AConfusion Rationale: Diphenhydramine and other first-generation H1 receptor antagonists may cause confusion (with impaired thinking, judgment, and memory), dizziness, hypotension, sedation, syncope, unsteady gait, and paradoxical central nervous system stimulation in older adults. Older adults may experience urinary retention, especially those with prostatic hypertrophy. Some of these adverse reactions derive from the anticholinergic effects of the drugs and are likely to be more severe if the patient is also taking other drugs with anticholinergic effects. Diphenhydramine is sometimes prescribed as a sleep aid for occasional use in older adults. As with many other drugs, smaller-than-usual dosages are indicated.

The nurse is preparing to administer newly prescribed intravenous phenytoin to a client. When reviewing the client's medical record, which prescription should the nurse question? AContinuous infusion of dextrose 5% in 0.9% saline B NPH insulin 40 units before meals CLabetalol 100 mg orally twice per day DKetorolac 15 mg IV push as needed for pain

AContinuous infusion of dextrose 5% in 0.9% saline Rationale: Phenytoin is not compatible with most IV fluids, especially those with dextrose. If the nurse observes a continuous infusion of a fluid that contains dextrose, they should understand that incompatibilities are likely and should not administer the medication as prescribed. Insulin, labetalol, and ketorolac do not have potential incompatibilities.

A nurse is reviewing the INR results for caring for a client who had a cerebral vascular accident and is receiving prescribed warfarin. The nurse notes the INR is 5.2. Which finding requires priority follow-up? AGum bleeding BGeneralized weakness CPharyngitis DAnorexia

AGum bleeding Rationale: Coumadin is an anticoagulant. The normal range of the INR for a client who is taking warfarin is 1 to 2.5 times normal, or 2-3. This INR level is elevated indicating the blood is taking longer to clot and presents a risk of internal bleeding. Generalized weakness post CVA is a normal finding. A sore throat (pharyngitis) and loss of appetite (anorexia) do not pose a serious risk at this time.

The nurse is caring for a female client who is requesting hormonal contraceptives. Which of the following questions should the nurse ask to assess for contraindications? AHave you ever had a blood clot? BHow many children do you have? CDo you drink alcohol? DDid you experience acne in adolescence?

AHave you ever had a blood clot? Rationale: A history of thromboembolic disorders is a contraindication to hormonal contraceptives; therefore, any history of thrombus should be assessed. The number of children/pregnancies and use of alcohol are probable history questions but are not contraindications to this method. Acne is a side effect of oral contraceptives but not a contraindication.

The nurse is providing discharge instructions to a client with a prescription for sublingual nitroglycerin. The nurse should inform the client to prepare for this most common side effect? AHeadache BDepression CDry mouth DAnorexia

AHeadache Rationale: Nitroglycerin is a potent vasodilator and a headache is the most common side effect. The headache comes on suddenly and can be severe, thus the client should be prepared for this effect. The other side effects listed are common side effects of oral medications, but not specifically to nitroglycerin.

The nurse is educating a client with end stage chronic obstructive pulmonary disease (COPD) about medication management. Which statement by the client indicates an understanding of the teaching? AI will use the albuterol in the nebulizer before my other inhalers each morning BI can use my tiotropium inhaler if I get short of breath CI will only use the fluticasone inhaler on the days I am really out of breath DThe side effects of these medications will be less severe because I'm not taking them by mouth

AI will use the albuterol in the nebulizer before my other inhalers each morning Rationale: Medication regimens used to treat COPD are based on disease severity. For grade III or IV (severe and very severe) COPD, medication therapy includes treatment with one or more bronchodilators and inhaled corticosteroids. Clients with COPD experience significant breathlessness and reduced FEV1 upon waking. Use of nebulized albuterol prior to administration of long-acting medications relaxes the airway and allows other medications to get deeper into the lungs. Tiotropium is a long-acting anticholinergic (muscarinic) and is not meant for rescue purposes. Fluticasone prevents inflammation and therefore, must be used every day. Clients with COPD will experience side effects of the medications due to the long duration of use.

The nurse is providing education to the client prescribed montelukast for the treatment of asthma. What medication should the nurse instruct the client to avoid? AIbuprofen BPrednisone CAmoxicillin DFormoterol

AIbuprofen Rationale: Montelukast should not be taken with NSAIDs. It increases the risk of bleeding as well as has the potential to make asthma symptoms worse. Prednisone, amoxicillin, and formoterol are all safe to administer to the client on montelukast.

The nurse is reviewing the prothrombin time results for a client who is taking warfarin. The nurse notes the value is 20 seconds. What is an appropriate nursing action? ARecognize that this is a therapeutic level. BAssess for bleeding gums or IV sites. CNotify the primary health care provider immediately. DObserve the client for hematoma development.

ARecognize that this is a therapeutic level. Rationale: For the client on warfarin therapy, this prothrombin level is within the therapeutic range. Therapeutic levels for warfarin are usually one and a half to two times the normal level.

A nurse is reviewing a client's medical history. The client has been newly diagnosed with hypertension and has been prescribed oral losartan as treatment. The nurse will clarify the use of losartan if which comorbidity is noted in the client's medical record? ARenal stenosis BHyperlipidemia CAtrial fibrillation DDiabetes

ARenal stenosis Rationale: Losartan is an angiotensin II receptor blocker used in the treatment of hypertension. Losartan is contraindicated in clients with renal stenosis due to the risk of kidney injury. Hyperlipidemia, atrial fibrillation, and diabetes are not known to be contraindicated in the use of losartan.

The nurse is collecting the health history of a client with heart disease who reports experiencing episodes of diarrhea. The client reports taking loperamide at home. Which of the following statements should the nurse make? ATaking this medication may increase your risk of an abnormal heart rhythm BIt is safe to drink alcohol while using this medication CUsing this medication may cause dependence DStop taking this medication if your symptoms do not improve by tomorrow

ATaking this medication may increase your risk of an abnormal heart rhythm Rationale: Loperamide decreases GI motility and is a nonprescription drug. It does not cause the central nervous system effects associated with opiate derivatives and lacks the potential for abuse. Loperamide should not be continued after 48 hours if improvement has not occurred. Loperamide has a black box warning because torsades de pointes, cardiac arrest, and death have been reported in people using higher than recommended dosages. Alcohol can increase the nervous system side effects of loperamide, such as dizziness, drowsiness, and difficulty concentrating.

Which medication is often contraindicated when taking warfarin? Atenolol Ferrous sulfate Chlorpromazine Acetylsalicylic acid

Acetylsalicylic acid Rationale Acetylsalicylic acid can cause decreased platelet aggregation, increasing the risk for undesired bleeding that may occur with administration of anticoagulants. It should not be administered unless specifically prescribed, usually by a cardiologist or other specialist, to manage serious risks of thrombosis. Ferrous sulfate does not affect warfarin; it is used for red blood cell synthesis. Atenolol is a beta-blocker that reduces blood pressure; it does not affect bleeding. Chlorpromazine is a neuroleptic; it does not affect bleeding.

The nurse administers albuterol to a 4-year-old child. Which intervention would assist the nurse in evaluating the effectiveness of this medication? Auscultate breath sounds. Collect a sputum sample. Conduct a neurological examination. Palpate chest excursion.

Auscultate breath sounds. Rationale Albuterol is an adrenergic medication that stimulates beta-receptors, leading to relaxation of the smooth muscles of the airway. The lungs should be auscultated to evaluate the effectiveness of this medication. Albuterol does not affect the consistency of pulmonary secretions. Albuterol will not cause central nervous system stimulation. Albuterol does not affect intercostal contractility; chest excursion is not the appropriate assessment.

To prevent excessive bruising when administering subcutaneous heparin, which technique will the nurse employ? Administer the injection via the Z-track technique. Avoid massaging the injection site after the injection. Use 2 mL of sterile normal saline to dilute the heparin. Inject the medication into the vastus lateralis muscle in the thigh.

Avoid massaging the injection site after the injection. Rationale The site of the injection should not be massaged to avoid dispersion of the heparin around the site and subsequent bleeding into the area. The Z-track technique and the intramuscular route are not used with heparin; subcutaneous injection and intravenous administration are the routes appropriate for heparin administration. The medication should be injected into the subcutaneous tissue slowly, not quickly. Diluting heparin with normal saline is unnecessary. Generally, heparin is provided by the pharmacy department in single-dose syringes.

The nurse is teaching a client with asthma about albuterol. How should the nurse best describe the action of this medication? A"The medication is given to reduce secretions that block airways." B"The medication will help to relax smooth muscles in the airways." C"The medication will stimulate the respiratory center in the brain." D"The medication will help to prevent pneumonia."

B"The medication will help to relax smooth muscles in the airways." Rationale: Albuterol is a bronchodilator and rescue drug of choice to treat asthma. It is a short-acting beta-adrenergic agonist that is used to prevent and treat wheezing, difficulty breathing, and chest tightness. Albuterol works by relaxing and opening the airways to make breathing easier. The medication comes as a tablet, syrup, inhaler and nebulizer. Albuterol does not reduce secretions, stimulate the respiratory center in the brain or prevent pneumonia.

The nurse is reviewing prescribed medications with a client. Which information should the nurse reinforce about captopril? ATake the medication with meals. BAvoid using salt substitutes. CRestrict fluids to 1000 mL/day. DAvoid green leafy vegetables.

BAvoid using salt substitutes. Rationale: Captopril is an angiotensin converting enzyme (ACE) inhibitor. It reduces aldosterone secretion, thereby reducing sodium and water retention. Captopril is used to treat hypertension and heart failure. Because it can cause an accumulation of serum potassium (i.e., hyperkalemia), clients should avoid the use of salt substitutes, which often contain potassium instead of sodium chloride. The other information does not apply to captopril.

A nurse is educating a client about the use of warfarin at home. The nurse should reinforce the need for the client to monitor which of the following? AExtended exposure to outdoor sunlight BConsistent intake of foods high in vitamin K CAvoidance of public transportation and large groups of people DLimit of strenuous physical exercise

BConsistent intake of foods high in vitamin K Rationale: Warfarin, an oral anticoagulant, works by causing a decrease in the vitamin K-dependent clotting factors produced by the liver. Due to this mechanism of action, vitamin K is used as the antidote for warfarin overdose. A diet high in vitamin K could counteract the therapeutic effect of warfarin. Foods high in vitamin K include dark green leafy vegetables, tomatoes, bananas, cheese and fish. Best practice no longer recommends limiting the intake of Vitamin K-containing foods, instead it is recommended to keep the intake of foods high in Vitamin K 'consistent'. The other actions do not pertain to warfarin.

Which food would the nurse encourage a client to eat while receiving treatment to prevent hypokalemia? Broccoli Oatmeal Fried rice Canned carrots

Broccoli Rationale Potassium is plentiful in green leafy vegetables; broccoli provides 207 mg of potassium per half cup. Oatmeal provides 73 mg of potassium per half cup. Rice provides 29 mg of potassium per half cup. Cooked fresh carrots provide 172 mg of potassium per half cup; canned carrots provide only 93 mg of potassium per half cup.

The nurse is preparing to administer an albuterol nebulizer treatment to a patient with asthma. Which assessment finding should be brought to the health care provider's attention prior to administering the medication? ATemperature of 101 deg F BHeart rate of 116 bpm CRespiratory rate of 28 DLower extremity edema

BHeart rate of 116 bpm Rationale: One of the more common adverse effects of beta-adrenergic medications, such as albuterol, is an increase in heart rate. Normal resting heart rate for children 10-years-old and older is the same as adults: 60 to 100 bpm. The nurse should report the heart rate to the health care provider prior to administering the medication.

The nurse is caring for a client with osteoporosis who has been prescribed alendronate. When providing care, which intervention would be a priority? AAdminister the alendronate 30 to 60 minutes before the client eats. BNotify the health care provider if the client reports jaw pain. CEncourage the client to increase their intake of vitamin D. DMonitor the client's serum calcium levels. Rationale: Alendronate is a bisphosphonate that helps slow down bone resorption, decreasing osteoporosis. Osteonecrosis of the jaw is a rare, adverse reaction to alendronate, and jaw pain can be a symptom of this. Therefore, notifying the health care provider of the jaw pain is the priority. The other interventions are also correct for a client with osteoporosis, but are not as important as reporting the potential adverse drug effect.

BNotify the health care provider if the client reports jaw pain. Rationale: Alendronate is a bisphosphonate that helps slow down bone resorption, decreasing osteoporosis. Osteonecrosis of the jaw is a rare, adverse reaction to alendronate, and jaw pain can be a symptom of this. Therefore, notifying the health care provider of the jaw pain is the priority. The other interventions are also correct for a client with osteoporosis, but are not as important as reporting the potential adverse drug effect.

A nurse has administered sublingual nitroglycerin to a client in the emergency department. Which clinical finding indicates an adverse response to the medication? APersistent chest pain BOrthostatic hypotension CDecreased heart rate DLabored breathing

BOrthostatic hypotension Rationale: Decreased blood pressure when changing positions is an unexpected response to nitroglycerin. The nurse should instruct the client to lay down and elevate the feet to promote venous return. Persistent chest pain is not an unexpected response. Additional doses may be required to alleviate angina. A side effect of nitroglycerin is tachycardia, not a decreased heart rate. Nitroglycerin is not associated with respiratory effects.

The nurse is evaluating the effectiveness of therapy for a client who received albuterol via nebulizer during an acute episode of shortness of breath due to asthma. Which finding is the best indicator that the therapy was effective? AAccessory muscle use has decreased. BOxygen saturation is greater than 90%. CRespiratory rate is 16 breaths/minute. DNo wheezes are audible.

BOxygen saturation is greater than 90%. Rationale: The goal for treatment of an asthma attack is to relieve bronchospasms and keep the oxygen saturation greater than 90%. Albuterol is a short-acting inhaled beta2-adrenergic agonist and the treatment of choice for an acute asthma attack. Pulse oximetry is an objective data point that the nurse should use to determine oxygenation status of the client. The other client data may occur when the client is too fatigued to continue with the increased work of breathing required in an asthma attack and, therefore, should not be used to evaluate effectiveness of treatment.

A client with chronic obstructive pulmonary disease (COPD) is receiving aminophylline 25 mg/hour intravenously (IV). Which finding would be associated with side effects of this medication? AFlushing and headache BRestlessness and palpitations CDecreased urine volume DPruritus

BRestlessness and palpitations Rationale: Aminophylline is a bronchodilator often used to treat symptom of asthma, bronchitis, and emphysema. Side effects include restlessness, palpitations, chest pain or discomfort, increased urine volume, vertigo, and vomiting. The other choices are not side effects of this drug. Question 11

The nurse is teaching the client how to properly use a dry powder capsule inhaler. How should the nurse instruct the client to use this type of inhaler? AShake inhaler before putting it in mouth BSeal lips tightly around mouthpiece and inhale rapidly and deeply CRinse mouthpiece in hot soapy water after using DBreathe in medicine slowly and deeply for about 3-5 seconds

BSeal lips tightly around mouthpiece and inhale rapidly and deeply Rationale: The client should breathe in quickly and deeply for up to 10 seconds when using a dry powder capsule inhaler. The client should not shake this type of inhaler. The mouthpiece can be rinsed with warm water but without soap or detergent.

The nurse is caring for a client after cardiac surgery who has been prescribed protamine sulfate. Which finding indicates that the treatment is having the intended effect? Question 6 Answer Choices AThe international normalized ratio (INR) is trending down. BThe bleeding from the surgical site has slowed. CThe client reports decreased chest pain. DThe respiratory rate is increased.

BThe bleeding from the surgical site has slowed.' Rationale: Protamine sulfate is the antidote for standard heparin and low molecular weight heparins (LMWHs). Protamine is typically given for bleeding that may not respond to merely withdrawing the heparin or when hemorrhaging is present. INR is used to determine the therapeutic level of warfarin (Coumadin), not heparin. Chest pain would be treated with nitroglycerin, but not protamine. The respiratory rate would be increased by naloxone if opiates were prescribed.

The nurse is providing discharge education to a client newly diagnosed with chronic obstructive pulmonary disease. The client is prescribed the diskus inhaler fluticasone propionate and salmeterol. The client asks, "How will I know when the inhaler is empty?" How should the nurse respond? AShake the canister to detect any fluid movement BThe number of doses that remain will be on the inhaler CDrop the canister in water to observe floating DEstimate how many doses are usually in the canister

BThe number of doses that remain will be on the inhaler Rationale: There are several methods to monitoring the contents of an inhaler. New MDIs such as diskus inhalers often have counters on them. The counters record the number of doses left in the canister. If the MDI does not have this feature, the client should write the date a refill is needed. This can be done directly on the canister in a permanent marker. Manufacturers do not recommend floating inhalers. The shaking or estimation method will not be accurate.

The nurse is discharging a client with a new prescription for tiotropium to help manage the symptoms of chronic obstructive pulmonary disease. What information should the nurse include in the discharge teaching? AIt may be a few days before you feel the full effects of tiotropium. BThis medication cannot be used to relieve sudden breathing problems. CBe sure to swallow the capsules with a full glass of water. DA common side effect is nausea and loose stools.

BThis medication cannot be used to relieve sudden breathing problems. Rationale: Tiotropium is a long-acting anticholinergic bronchodilator. The medication comes as a capsule to use with a specially designed inhaler - clients should never swallow the capsules. For new prescriptions, it's important to tell the client that they may start breathing better with the full dose but it may take a few weeks to feel the full effects. It cannot be used as a fast-acting inhaler. Due to its anticholinergic properties, it may cause constipation (not loose stools).

690) A client was prescribed furosemide. The nurse would instruct the client to include which food in the diet? Liver Apples Cabbage Bananas

Bananas Rationale Furosemide is a loop diuretic that increases potassium excretion by preventing renal absorption. Bananas have a significant amount of potassium. Bananas: 450 mg; cabbage: 243 mg; liver: 73.6 mg; apples: 100 to 120 mg.690) A client was prescribed furosemide. The nurse would instruct the client to include which food in the diet? Liver Apples Cabbage *Bananas Rationale Furosemide is a loop diuretic that increases potassium excretion by preventing renal absorption. Bananas have a significant amount of potassium. Bananas: 450 mg; cabbage: 243 mg; liver: 73.6 mg; apples: 100 to 120 mg.

Oral contraceptives are prescribed for a client who smokes heavily. Which side effect would the nurse warn the client might occur? Blood clots Cervical cancer Ovarian cancer Risk of coronary heart disease later in life

Blood clots Rationale Heavy smoking is a major risk factor for an increased risk of thrombosis or blood clots. Cervical cancer is associated with human papillomavirus infection, not oral contraceptive use. Oral contraceptives have a protective effect against ovarian cancer. Although there is an increased risk of coronary heart disease while taking an oral contraceptive, this risk abates when it is no longer taken and does not carry over into later life.

A 31-year-old client is seeking contraceptive information. While obtaining the client's history, which factor indicates to the nurse that oral contraceptives are contraindicated? Older than 30 years Current hypothyroidism Two multiple pregnancies Blood pressure 162/110

Blood pressure 162/110 Rationale Oral contraceptives may cause or exacerbate hypertension; even borderline hypertension places the client at risk for a brain attack. Oral contraceptives are not contraindicated for women older than 30 years of age if there are no known risk factors. There is no relationship between oral contraceptives and multiple births. Contraceptives are not contraindicated in clients who have hypothyroidism.

A client with systemic lupus erythematosus is taking prednisone. Which foods would the nurse encourage the client to eat while receiving treatment to prevent hypokalemia? Broccoli Oatmeal Fried rice Cooked carrots .

Broccoli Rationale Potassium is plentiful in green leafy vegetables; broccoli provides 207 mg of potassium per half cup. Oatmeal provides 73 mg of potassium per half cup. Rice provides 29 mg of potassium per half cup. Cooked fresh carrots provide 172 mg of potassium per half cup; canned carrots provide only 93 mg of potassium per half cup

The nurse is administering 40 mg of furosemide intravenously. Which sensation reported by the client would the nurse consider when determining that it is being administered too quickly? Full bladder Buzzing ears Fast heartbeat Numb arms and legs

Buzzing ears Rationale Rapid administration of furosemide can cause tinnitus (a perceived ringing or buzzing in the ears), loss of hearing, and ear pain. Furosemide has a diuretic effect, so a full bladder with a need to urinate is an anticipated response unrelated to speed of administration. Furosemide does not affect the heart rate. Furosemide does not cause peripheral neuropathy.

A client is prescribed alendronate. Which instruction should the nurse emphasize when teaching about this medication? A"Take the medication with a full glass of milk two hours after meals." B"It is recommended that you take this medication with calcium and a glass of juice." C"Be sure to take this medication on an empty stomach." D"You may take this medication after any meal, at the same time every day."

C"Be sure to take this medication on an empty stomach." Rationale: Alendronate (Fosamax) is used to treat and prevent osteoporosis. It should be taken first thing in the morning with 6 to 8 ounces of plain water at least 30 minutes before other medication or food. Food and fluids (other than water) greatly decrease the absorption of this medication. The client must also be instructed to remain in the upright position for 30 minutes following the dose to facilitate passage into the stomach and minimize irritation of the esophagus

A 42-year-old male client diagnosed with hypertension tells the nurse he no longer wants to take the prescribed propranolol. Which client statement best explains the reason why he does not want to take this medication? A"I have difficulty falling asleep." B"I'm having problems with my stomach." C"I'm experiencing decreased sex drive." D"I feel so tired all the time."

C"I'm experiencing decreased sex drive." Rationale: Propranolol is a beta-blocker used to treat many conditions, such as essential tremors, angina, hypertension and heart rhythm disorders. Common side effects of this drug include nausea, diarrhea, constipation, stomach cramps, rash, tiredness, dizziness, sleep problems and vision changes. Additionally, propranolol may cause decreased sex drive, impotence or difficulty having an orgasm in men. The clients can be switched to an alternative antihypertensive, such as an angiotensin-converting enzyme (ACE) inhibitor or a calcium channel blocker.

The inpatient hospital nurse is caring for a client with hypokalemia. The health care provider prescribed a potassium intravenous (IV) infusion of 40 mEq potassium chloride in 250 mL normal saline to be infused over 4 hours. The nurse receives the infusion from the pharmacy. Which action should the nurse take next? AConfirm patency of the peripheral venous access device and start the infusion BNotify the health care provider of the inappropriate dose of the prescribed IV potassium CAsk another nurse to verify the prescription, IV solution and serum potassium level DAsk another nurse to witness the addition of the prescribed potassium to the IV solution

CAsk another nurse to verify the prescription, IV solution and serum potassium level Rationale: Since potassium chloride is considered a high alert medication, especially when given IV, having two nurses verify the order and IV bag is recommended. The nurses should compare the supplied IV bag to the prescriber's order. If potassium IV is infused too rapidly or in too high a dose, it can cause dysrhythmias and cardiac arrest. In addition, the second nurse should also verify the client's most recent serum potassium level to ensure that the prescription is appropriate. The prescribed dose and amount of IV solution is within normal range for IV potassium replacement therapy. Potassium should never be added by a nurse to an IV bag.

The client has been treated with long-term glucocorticoid therapy. While completing the physical assessment, which finding should the nurse expect? AJaundice BPeripheral edema CBuffalo hump DIncreased muscle mass

CBuffalo hump Rationale: The most common side effects of glucocorticoid therapy include increased appetite including weight gain, increased blood glucose, acne, thinning of the skin, easy bruising and change in body shape (increase in fatty tissue on the trunk with thinner legs and arms). The client may also develop a hump behind the shoulders due to the accumulation of fat on the back of the neck. This is referred to as a buffalo hump. Jaundice, peripheral edema and increased muscle mass are not side effects of glucocorticoid therapy.

The nurse is teaching a client about newly prescribed inhaled budesonide. The nurse should teach the client to report which finding to the healthcare provider? ARounded face BBradycardia CIncreased thirst DCough

CIncreased thirst Question Explanation Rationale: Respiratory disorders, such as asthma, status asthmatic, chronic obstructive pulmonary disease (COPD), and rhinitis, may all be treated with corticosteroids, including budesonide. Corticosteroids have many common side effects including cushingoid features, such as "moon face" due to redistribution of fat. Fluid retention is also common when using corticosteroids. Increased thirst may be an indication of hyperglycemia and should be reported. Corticosteroids can increase heart rate. A cough is normal with corticosteroids as the airway is dilated.

A client who is 34 weeks gestation is diagnosed with a pulmonary embolism. Which of these medications should the nurse plan to administer? AOral low-dose aspirin BOral warfarin CIntravenous heparin DSubcutaneous enoxaparin .

CIntravenous heparin Rationale: Clients diagnosed with pulmonary embolism (PE), whether pregnant or not, are initially treated with intravenous unfractionated heparin. Alternatively, low molecular weight heparin such as enoxaparin can be used to treat women who are pregnant. Warfarin should never be given during pregnancy due to its teratogenic effects. Although aspirin has anticoagulant properties, low-dose aspirin therapy (81 mg) is more often used prophylactically, not for the treatment of a PE

The nurse is caring for a client who is being treated for heart failure. After completing the medication reconciliation process, the nurse notes that the prescriber has added lisinopril 5mg orally bid. Which medication from the list below should the nurse question due to possible drug-to-drug interaction with lisinopril? Medication Administration Record Name Dose Route Frequency Metoprolol 50 mg Orally BID Glipizide 5 mg Orally Daily Naproxen 200 mg Orally TID Enoxaparin 40 mg SQ Q12H AMetoprolol BGlipizide CNaproxen DEnoxaparin

CNaproxen Question Explanation Rationale: Nonsteroidal anti-inflammatory (NSAIDs) drugs, such as naproxen, reduce the antihypertensive effects of angiotensin converting enzyme (ACE) inhibitors such as lisinopril. The use of NSAIDs and ACE inhibitors may also predispose patients to develop acute renal failure. Additionally, naproxen increases the risk of heart attack or stroke with heart disease. The nurse should clarify the naproxen with the health care provider. The other medications are not known to interact with lisinopril.

The nurse is providing teaching to the client prescribed albuterol for the management of asthma. The nurse is including reportable side effects in the teaching plan. Which of the following side effects is the priority? ANervousness BHeadache CPalpitations DMuscle aches

CPalpitations Rationale: Side effects of albuterol include nervousness, shakiness, headache, throat irritation, and muscle aches. Muscle tremor is the most frequent adverse effect. The main risks with adrenergic bronchodilators, particularly in older adults, are excessive cardiac and central nervous system (CNS) stimulation. Symptoms of cardiac stimulation include angina, tachycardia, and palpitations. Symptoms of central nervous system (CNS) stimulation consist of agitation, anxiety, insomnia, seizures, and tremors. Other reported effects may include serious dysrhythmias and cardiac arrest.

The client who was admitted with exacerbation of ulcerative colitis has developed hyperglycemia. Which medication that the client was prescribed most likely caused this adverse drug effect? ADicyclomine BAcetaminophen CPrednisone DDiphenoxylate/atropine

CPrednisone Rationale: Prednisone is a corticosteroid, specifically a glucocorticoid. Corticosteroid therapy may be prescribed during exacerbations of ulcerative colitis to decrease inflammation. Common adverse effects include hyperglycemia, osteoporosis, peptic ulcer disease and an increased risk for infection. The nurse should monitor clients who are receiving prednisone for hyperglycemia. Dicyclomine hydrochloride and diphenoxylate with atropine are cholinergic blocking drugs prescribed for gas (flatus) and diarrhea, commonly seen with ulcerative colitis. Acetaminophen is a non-narcotic analgesic given for mild-to-moderate pain. None of those drugs are known to cause an elevated blood sugar.

The nurse on a cardiac unit is caring for a client who is receiving nitroglycerin intravenously for unstable angina. During administration of the medication, which assessment is the priority? ARespiratory rate BCardiac enzymes CCardiac rhythm DBlood pressure

DBlood pressure Rationale: Nitroglycerin is a drug that is used to provide relief from myocardial chest pain and treat hypertensive emergencies. Nitroglycerin causes vasodilation. Common adverse effects of nitroglycerin include hypotension, headache and dizziness; therefore, monitoring the client's blood pressure is the priority. Nitroglycerin does not affect respirations, cardiac enzyme levels or heart rhythm.

The nurse is monitoring the client who is taking newly prescribed antihypertensive medication. Which finding should indicate to the nurse that the client might be experiencing an allergic reaction to the medication? AMild decrease in blood pressure BIncreased urine output CLeft-sided weakness DDevelopment of a rash

DDevelopment of a rash Rationale: Allergic reactions are often manifested by the presence of a rash, urticaria, gastrointestinal symptoms, and itching. A mild decrease in blood pressure is the intended effect of the medication. Increased urinary output and unilateral weakness are not indications of an allergic reaction.

A nurse is assessing a client receiving intravenous potassium chloride. The client verbalizes pain to the IV site. The site appears swollen and is warm to touch. Which action does the nurse perform? ADecrease the rate of the infusion BApply ice to the IV access site CInform the client that this is an expected finding DDiscontinue the IV catheter

DDiscontinue the IV catheter Rationale: The nurse should discontinue the IV catheter. The client's symptoms are indicative of phlebitis, inflammation of the vein. Decreasing the rate of the infusion will not treat the swelling or injury to the vein. Applying ice to the access site does not address the possible vein injury caused by the medication. Pain, swelling, and warmth are not expected findings for a patent IV access site.

The nurse is caring for a client prescribed warfarin therapy for treatment of persistent atrial fibrillation. Which of the following may potentiate the effect of this medication? ASt. John wort BEstrogen CVitamin K DGreen tea

DGreen tea Question Explanation Rationale: Warfarin, an anticoagulant agent used to prevent thrombosis and risk of stroke in clients with atrial fibrillation, is associated with many drug and food interactions. Careful assessment with a pharmacist/formulary is recommended to avoid potential complications. Green tea can potentiate the effect of warfarin and increase bleeding. St. John's wort, estrogen, and vitamin K may inhibit the action requiring higher doses of the anticoagulant.

The nurse is caring for a client prescribed warfarin therapy for treatment of persistent atrial fibrillation. Which of the following may potentiate the effect of this medication? ASt. John wort BEstrogen CVitamin K DGreen tea

DGreen tea Rationale: Warfarin, an anticoagulant agent used to prevent thrombosis and risk of stroke in clients with atrial fibrillation, is associated with many drug and food interactions. Careful assessment with a pharmacist/formulary is recommended to avoid potential complications. Green tea can potentiate the effect of warfarin and increase bleeding. St. John's wort, estrogen, and vitamin K may inhibit the action requiring higher doses of the anticoagulant.

A client is being discharged with a prescription for warfarin. Which information is most important to be included in the nurse's discharge teaching? ATake acetaminophen for minor pain BUse a soft toothbrush CAvoid eating leafy green vegetables DReport nose or gum bleeding

DReport nose or gum bleeding Rationale: The most important teaching is to make sure that the client understands to report any sign of bleeding including nose or gum bleeding, blood noted in stools or urine, coughing up blood, or easy bruising. Dark green leafy vegetables are high in vitamin K which can lower the effectiveness of warfarin (Coumadin). Acetaminophen does not contain aspirin which can cause internal bleeding so is safe to use when taking warfarin. A soft toothbrush will be less irritating to the gums and therefore decrease the risk of bleeding gums. Although green leafy vegetables contain Vitamin K, it is no longer recommended to avoid them but to keep their intake consistent.

The nurse is preparing to administer prescribed warfarin to a client with a mechanical heart valve. Which finding should the nurse report to the healthcare provider? AThe INR is 3.0. BThe peripheral IV site has been oozing blood. CThe aPTT is 30. The aPTT is 30. DThe client has cola-colored urine.

DThe client has cola-colored urine. Rationale: Cola-colored urine is a sign of hematuria. This may be caused by the warfarin or a sign of another problem. It is common to have oozing around IV sites in clients on anticoagulants. The INR of 3.0 is an expected finding. The aPTT should not be affected by warfarin and is also an expected finding.

A client is prepared for a supratentorial craniotomy with burr holes, and an intravenous infusion of mannitol is instituted. The nurse concludes that this medication is primarily given for which purpose? Lower blood pressure Prevent hypoglycemia Increase cardiac output Decrease fluid in the brain

Decrease fluid in the brain Rationale Osmotic diuretics remove excessive cerebrospinal fluid (CSF), reducing intracranial pressure. Osmotic diuretics increase, not decrease, the blood pressure by increasing the fluid in the intravascular compartment. Osmotic diuretics do not directly influence blood glucose levels. Although there is an increase in cardiac output when the vascular bed expands as CSF is removed, it is not the primary purpose of administering the medication.

Which response to fludrocortisone will the nurse teach a client with adrenal insufficiency to report? Select all that apply. One, some, or all responses may be correct. Edema Rapid weight gain Fatigue in the afternoon Unpredictable changes in mood Increased frequency of urination

Edema Rapid weight gain Rationale Fludrocortisone has a strong effect on sodium retention by the kidneys, which leads to fluid retention, causing edema and weight gain. Fatigue may occur with adrenal insufficiency and is not related to cortisone therapy. Unpredictable changes in mood commonly occur but are not as serious a threat as fluid retention. Fluid retention, and thus decreased urination, may occur.

After the nurse provides education about hydrochlorothiazide, the client will agree to notify the health care provider regarding the development of which symptom? Insomnia Nasal congestion Increased thirst Generalized weakness

Generalized weakness Rationale Generalized weakness is a symptom of significant hypokalemia, which may be a sequela of diuretic therapy. Insomnia is not known to be related to hypokalemia or hydrochlorothiazide therapy. Although a stuffy nose is unrelated to hydrochlorothiazide therapy, it can occur with other antihypertensive medications. Increased thirst is associated with hypernatremia. Because this medication increases excretion of water and sodium in addition to potassium and chloride, hyponatremia, not hypernatremia, may occur.

Which conditions would the nurse identify as decreasing the effectiveness of estrogen therapy? Select all that apply. One, some, or all responses may be correct. Habit of smoking Use of anticoagulants Use of tricyclic antidepressants Presence of endometrial cancer Presence of thromboembolic disorders

Habit of smoking Use of anticoagulants Rationale The effectiveness of estrogen therapy decreases with smoking and decreases with the use of anticoagulants. The use of tricyclic antidepressants along with estrogen may result in medication toxicity. Estrogen therapy is not recommended in clients with endometrial cancer and thromboembolic disorders because it may increase the risk of these complications.

The nurse would instruct a client to stop taking an oral contraceptive and notify the health care provider immediately for the presence of which clinical findings? Select all that apply. One, some, or all responses may be correct. Nausea Headaches Weight loss Visual disturbances Increased menstrual flow

Headaches Visual disturbances Rationale Headaches, either sudden or persistent, may indicate hypertension or a cardiovascular event. Visual disorders, such as partial or complete loss of vision or double vision, may indicate neuro-ocular lesions, which are associated with the use of some oral contraceptives. Nausea is a side effect that is not life threatening; the dose may need to be adjusted or the product changed to an alternative. Weight gain, not loss, may occur because of fluid retention. Menorrhagia is less likely to occur.

A client with a partial occlusion of the left common carotid artery is to be discharged while still receiving warfarin. Which adverse effect will the nurse identify as a reason for the client to seek medical consultation? Select all that apply. One, some, or all responses may be correct. Hematuria Hemoptysis Delayed clotting from minor cuts and scrapes Bleeding from gums when brushing teeth Vomiting coffee-ground emesis

Hematuria Hemoptysis Vomiting coffee-ground emesis Rationale Warfarin causes an increase in the prothrombin time and International Normalized Ratio (INR) level, leading to an increased risk for bleeding. Any abnormal or prolonged bleeding must be reported, because it may indicate an excessive level of the medication. Common side effects including bruising, delayed clotting and bleeding gums do not require immediate intervention. However, hematuria and hemoptysis are evidence of more serious bleeding and require immediate attention. Coffee-ground emesis is a sign of gastric bleeding.

A client develops a deep vein thrombophlebitis in her leg 3 weeks after giving birth and is admitted for anticoagulant therapy. The nurse would anticipate developing a teaching plan for which anticoagulant? Heparin Warfarin Clopidogrel Enoxaparin

Heparin Rationale Heparin is the medication of choice during the acute phase of a deep vein thrombosis; it prevents conversion of fibrinogen to fibrin and of prothrombin to thrombin. Warfarin, a long-acting oral anticoagulant, is started after the acute stage has subsided; it is continued for 2 to 3 months. Clopidogrel is a platelet aggregate inhibitor and is used to reduce the risk of a brain attack. A low-molecular-weight heparin (e.g., enoxaparin) is not administered during the acute stage; it may be administered later to prevent future deep vein thromboses.

Valsartan, an angiotensin II receptor antagonist, is prescribed for a client. The nurse will monitor the client for which adverse effect? Constipation Hyperkalemia Hypertension Change in visual acuity

Hyperkalemia Rationale Hyperkalemia may occur with valsartan. Angiotensin II receptor antagonists, such as valsartan, block vasoconstrictor and aldosterone-producing effects of angiotensin II at receptor sites to decrease blood pressure. Hypotension, not hypertension, may occur. Diarrhea, not constipation, may occur with valsartan. Valsartan does not cause altered visual acuity.

Which adverse medication effect will a nurse monitor for in a client with cirrhosis of the liver who develops ascites and is prescribed spironolactone? Bruising Tachycardia Hyperkalemia Hypoglycemia

Hyperkalemia Rationale Spironolactone is a potassium-sparing diuretic that is used to treat clients with ascites; therefore the nurse would monitor the client for signs and symptoms of hyperkalemia. Bruising and purpura are associated with cirrhosis, not with the administration of spironolactone. Spironolactone does not cause tachycardia. Spironolactone does not cause hypoglycemia.

Which instruction would the nurse give an unlicensed assistive personnel (UAP) to perform while caring for a client prescribed captopril? Select all that apply. One, some, or all responses may be correct. Obtain blood pressure. Measure intake and output. Weigh the client every morning. Notify the nurse if the client has a dry cough. Assist the client to change positions slowly.

Obtain blood pressure. Measure intake and output. Weigh the client every morning. Notify the nurse if the client has a dry cough. Assist the client to change positions slowly. Rationale ACE inhibitors such as captopril are prescribed for the management of hypertension, heart failure, and diabetic nephropathy. The nurse would ask the UAP caring for a client taking captopril to perform several tasks. The UAP would obtain the client's blood pressure. The UAP would also measure the client's intake and output as well as obtain a daily weight in the morning. This data would help the nurse determine the client's fluid volume status and is an important component of heart failure management. The UAP would be aware that a dry cough is a common side effect of ACE inhibitors. Because of the blood pressure-lowering effects of this medication, the nurse would instruct the UAP to assist the client to make sure the client changes positions slowly.

A child is prescribed intravenous mannitol. The nurse understands mannitol belongs to which classification of diuretics? Loop Osmotic Potassium sparing Carbonic anhydrase inhibitor

Osmotic Rationale Osmotic diuretics, such as mannitol, increase the osmotic pressure of glomerular filtrate and thus decrease absorption of sodium; they are used to treat cerebral edema and increased intraocular pressure. Loop diuretics, such as furosemide, inhibit resorption of sodium and potassium in the loop of Henle; they are used for heart failure and pulmonary edema. Potassium-sparing diuretics, such as spironolactone, interfere with sodium resorption in the distal tubules, thus decreasing potassium excretion; they are used to treat cirrhotic ascites and pulmonary edema. Carbonic anhydrase inhibitors, such as acetazolamide, increase sodium excretion by decreasing sodium-hydrogen ion exchange. They are used to treat seizure disorders and open-angle glaucoma.

Which information from the client's history would the nurse identify as a risk factor for developing osteoporosis? Takes estrogen therapy Receives long-term steroid therapy Has a history of hypoparathyroidism Engages in strenuous physical activity

Receives long-term steroid therapy Rationale Increased levels of steroids will accelerate bone demineralization. Hyperparathyroidism, not hypoparathyroidism, accelerates bone demineralization. Weight-bearing that occurs with strenuous activity promotes bone integrity by preventing bone demineralization. Estrogen promotes deposition of calcium into bone which may prevent, not cause, osteoporosis.

Which action by a client taking alendronate requires correction? Select all that apply. One, some, or all responses may be correct. Taking medication twice a week Taking medication before rising Taking medication with breakfast Taking medication before bedtime Taking medication with apple juice

Taking medication twice a week Taking medication before rising Taking medication with breakfast Taking medication before bedtime Taking medication with apple juice Rationale Bisphosphonates such as alendronate have specific dosing instructions. The medication is given once a week. Clients must remain in an upright position for 30 minutes after taking the medication. Clients should refrain from eating right before they are given the medication and should take it with a full glass of water because of the risk for esophagitis. Alendronate must be administered in the morning. Clients should take the medication with water.

The nurse has administered sublingual nitroglycerin. Which outcome would the nurse use to determine the effectiveness of sublingual nitroglycerin? Relief of anginal pain Improved cardiac output Decreased blood pressure Ease in respiratory effort

Relief of anginal pain Rationale Cardiac nitrates relax smooth muscles of the coronary arteries; they dilate and deliver more blood to heart muscle, relieving ischemic pain. Although cardiac output may improve because of improved oxygenation of the myocardium, improved cardiac output is not a basis for evaluating the effectiveness of sublingual nitroglycerin. Although dilation of blood vessels and a subsequent drop in blood pressure is a reason why intravenous (IV) nitroglycerin may be administered, decreased blood pressure is not the basis for evaluating the effectiveness of sublingual nitroglycerin, which is indicated for pain relief. Although superficial vessels dilate, lowering the blood pressure and creating a flushed appearance, dilation of superficial blood vessels is not the basis for evaluating the medication's effectiveness.

Which pain characteristic would the nurse expect to observe when a client is experiencing anginal pain? Unchanged by rest Precipitated by light activity Described as a knifelike sharpness Relieved by sublingual nitroglycerin

Relieved by sublingual nitroglycerin Rationale Relief by sublingual nitroglycerin is a classic reaction because it causes vasodilation of peripheral veins and arteries, thereby decreasing oxygen demand by decreasing preload. To a lesser extent, sublingual nitroglycerin dilates coronary arteries, which increases oxygen to the myocardium, thereby decreasing pain. Immediate rest frequently relieves anginal pain. Angina usually is precipitated by exertion, emotion, or a heavy meal. Angina usually is described as tightness, indigestion, or heaviness.

Which finding in a menopausal client's health history would prevent the health care provider from prescribing hormone replacement therapy? Select all that apply. One, some, or all responses may be correct. Smoking Cirrhosis Cholecystitis Breast cancer Deep vein thrombosis

Smoking Cirrhosis Cholecystitis Breast cancer Deep vein thrombosis Rationale Use of estrogens can have major side effects, especially if the client smokes. The nurse would provide information to the client about smoking cessation. Clients with cirrhosis have a decreased ability to break down medications, especially estrogen. Cholecystitis can worsen in clients taking estrogen. Clients at risk for breast and endometrial cancer should not take estrogen because it can further increase the risk. Estrogens can lead to deep vein thrombosis.

Which outcome would the nurse expect when caring for a child receiving adrenocorticosteroid therapy? Accelerated wound healing Development of hyperkalemia Increased antibody production Suppressed inflammatory process

Suppressed inflammatory process Rationale Because of the suppression of the inflammatory process, the nurse must be alert to the subtle symptoms of infection, such as changes in appetite, sleep patterns, and behavior. Adrenocorticosteroid therapy delays (not accelerates) wound healing. Adrenocorticosteroid therapy may cause hypokalemia, not hyperkalemia, because of the accompanying retention of sodium and fluid. Adrenocorticosteroid therapy decreases (not increases) the production of antibodies.

Which information will the nurse include when teaching a client about potassium chloride effervescent tablets? Chew the tablet completely. Take the medication with food. Take the medication at bedtime. Use warm water to dissolve the tablet.

Take the medication with food. Rationale Eating food when taking the medication will decrease gastrointestinal irritation. Side effects of this medication include abdominal cramps, diarrhea, and ulceration of the small intestine. Chewing the tablet completely will cause oral mucosal irritation and is not the way the medication should be administered. Taking the medication at bedtime increases the possibility of mucosal irritation because the gastrointestinal tract is empty during the night. The tablet should be dissolved in cold water or juice to make it more palatable.

Upon admission to the emergency center, an adult client with acute status asthmaticus is prescribed this series of medications. In which order should the nurse administer the prescribed medications? (Arrange from first to last.) I A. Prednisone (Deltasone) orally. B. Gentamicin (Garamycin) IM. C. Albuterol (Proventil) puffs. D. Salmeterol (Serevent Diskus).

The correct order is: E. Albuterol (Proventil) puffs. F. Salmeterol (Serevent Diskus). G. Prednisone (Deltasone) orally. H. Gentamicin (Garamycin) IM. Status asthmaticus is potentially a life-threatening respiratory event, so albuterol, a beta2 adrenergic agonist and short acting bronchodilator, should be administered by inhalation first to provide rapid and deep topical penetration to relieve bronchospasms, dilate the bronchioles, and increase oxygenation. In stepwise management of persistent asthma, a long-action bronchodilator, such as salmeterol (Serevent Diskus), with a 12-hour duration of action should be given next. Prednisone, an oral corticosteroid, provides prolonged anti-inflammatory effects and should be given after the client's respiratory distress begins to resolves. Gentamicin, an antibiotic, is given deep IM, which can be painful, and may require repositioning the client, so should be last in the sequence.

A client who has a habit of smoking is on estrogen therapy. Which condition is the client at most risk of developing? Thrombosis Gastrointestinal upset Risk of developing cancer Decreased effectiveness of estrogen

Thrombosis Rationale Smoking along with estrogen therapy may increase the risk of thrombosis. Estrogen taken on an empty stomach may cause gastrointestinal upset. Estrogen is not prescribed to clients with endometrial or breast cancer. The effectiveness of estrogen decreases with the use of anticoagulants, rifampicin, and St. John wort.

664) A client is prescribed albuterol to relieve severe asthma. Which adverse effects will the nurse instruct the client to anticipate? Select all that apply. One, some, or all responses may be correct. -Tremors -Lethargy -Palpitations -Bronchoconstriction -Decreased pulse rate

Tremors Palpitations Rationale Albuterol's sympathomimetic effect causes central nervous system (CNS) stimulation, precipitating tremors, tachycardia, and palpitations. Lethargy is an adverse effect of medications that cause CNS depression, not CNS stimulation. Albuterol causes bronchodilation, not bronchoconstriction. Albuterol will cause tachycardia, not bradycardia.

A client with chronic liver disease reports, 'My gums have been bleeding spontaneously.' The nurse identifies small hemorrhagic lesions on the client's face. The nurse concludes that the client needs which additional supplement? Bile salts Folic acid Vitamin A Vitamin K

Vitamin K Rationale Fat-soluble vitamin K is essential for synthesis of prothrombin by the liver; a lack results in hypoprothrombinemia, inadequate coagulation, and hemorrhage. Although cirrhosis may interfere with production of bile, which contains the bilirubin needed for optimum absorption of vitamin K, the best and quickest manner to counteract the bleeding is to provide vitamin K intramuscularly. Folic acid is a coenzyme with vitamins B 12 and C in the formation of nucleic acids and heme; thus a deficiency may lead to anemia, not bleeding. Vitamin A deficiency contributes to the development of polyneuritis and beriberi, not hemorrhage.

Which response would a nurse monitor for when a client is receiving furosemide to relieve edema? Select all that apply. One, some, or all responses may be correct. Weight loss Negative nitrogen balance Increased urine specific gravity *Excessive loss of potassium ions Pronounced retention of sodium ions

Weight loss Rationale Each liter of fluid weighs 2.2 pounds (1 kilogram). Assessing weight loss is an objective measure of the effectiveness of the medication. Furosemide is a potent diuretic that is used to provide rapid diuresis in clients with pulmonary edema; it acts in the loop of Henle and causes depletion of electrolytes, such as potassium and sodium. A negative nitrogen balance would not be monitored. Furosemide does not affect protein metabolism. With increased fluid loss, the specific gravity is typically decreased.

A client is receiving furosemide to relieve edema. The nurse will monitor the client for which responses? Select all that apply. One, some, or all responses may be correct. Weight loss Negative nitrogen balance Increased urine specific gravity Excessive loss of potassium ions Pronounced retention of sodium ions

Weight loss Excessive loss of potassium ions Rationale Each liter of fluid weighs 2.2 pounds (1 kilogram). Assessing weight loss is an objective measure of the effectiveness of the medication. Furosemide is a potent diuretic that is used to provide rapid diuresis in clients with pulmonary edema; it acts in the loop of Henle and causes depletion of electrolytes, such as potassium and sodium. A negative nitrogen balance does not affect protein metabolism. With increased fluid loss, the specific gravity is likely to be lowered. Furosemide inhibits the reabsorption of sodium.

Which statement about appropriate foods to consume when taking warfarin would indicate that the client needs further teaching? -"Eggs provide a good source of iron, which is needed to prevent anemia." -"Yellow vegetables are high in vitamin A and should be included in the diet." -"Dark green leafy vegetables are high in vitamin K, so I should eat them more often." -"Milk and other high-calcium dairy products are necessary to counteract bone density loss."

should be included in the diet." -"Dark green leafy vegetables are high in vitamin K, so I should eat them more often." Rationale Foods high in vitamin K should be limited to the usual amounts eaten by the client when establishing the prothrombin time/international normalized ratio because vitamin K is part of the body's blood-clotting mechanism and will counter the effects of warfarin if eaten in excess. Foods containing protein and iron are permitted because they are unrelated to blood clotting. Foods containing vitamin A are permitted because vitamin A is unrelated to blood clotting. Foods containing calcium are permitted because calcium is unrelated to blood clotting.

Which statements would the nurse include when teaching a client who is considering medroxyprogesterone injection as a form of hormonal birth control? Select all that apply. One, some, or all responses may be correct. -It lasts for 3 months. -It can cause nausea and vomiting. -It contains both progestin and estrogen. -It decreases the risk of age-related loss of bone density. -Side effects include drowsiness or insomnia and depression. -The most severe undesirable effects are liver dysfunction and thromboembolic disorders.

*It lasts for 3 months. *It can cause nausea and vomiting. *Side effects include drowsiness or insomnia and depression. *The most severe undesirable effects are liver dysfunction and thromboembolic disorders. Rationale Medroxyprogesterone does last for 3 months. It can cause nausea and vomiting, as well as lethargy or insomnia and depression, although the most undesirable severe effects are liver dysfunction and thromboembolic disorders, including thrombophlebitis, cerebrovascular disorder, pulmonary embolism, and retinal thrombosis. Medroxyprogesterone does not contain both progestin and estrogen; it is progestin-only. Medroxyprogesterone does not decrease the risk of age-related loss of bone density; in fact, it has a black-box warning because it can cause bone mineral density loss that puts women at higher risk for osteoporosis and bone fractures later in life.

A client receiving fluphenazine decanoate develops dystonia early in therapy. Which medication would the nurse anticipate administering to reverse this side effect? -Nafarelin -Fluoxetine -Trandolapril -Benztropine

-Benztropine Rationale: Dystonia is an extrapyramidal side effect (EPS) of fluphenazine decanoate. The anticholinergic benztropine is used to reverse the signs and symptoms (e.g., oculogyric crisis, torticollis, retrocollis) of dystonia. Nafarelin is a gonadotropin that stimulates the release of luteinizing hormone and follicle-stimulating hormone. Fluoxetine is a selective serotonin reuptake inhibitor antidepressant. Trandolapril is an angiotensin-converting enzyme inhibitor antihypertensive.

773) The health care provider prescribes neostigmine for a client with myasthenia gravis. Which client statement indicates understanding regarding medication management plans? - 'I must keep the medication in a container in the refrigerator.' - I should take the medication at the exact time that is listed on the prescription.' -'I will plan to take the medication between meals.' -'I expect that the onset of the medication's action will occur several hours after I take it.'

- I should take the medication at the exact time that is listed on the prescription.' Rationale: Neostigmine should be taken as prescribed, usually before meals, to limit dysphagia and possible aspiration. Keeping neostigmine refrigerated is not necessary; it may be kept at room temperature. Neostigmine should be taken with milk to prevent gastrointestinal irritation; usually it is taken about 30 minutes before meals. The onset of the action of neostigmine occurs 45 to 75 minutes after administration; the duration of its action is 2.5 to 4 hours.

Which adverse response would a nurse assesses for when carbidopa-levodopa is prescribed for a client with Parkinson disease? Select all that apply. One, some, or all responses may be correct. -Nausea -Lethargy -Bradycardia -Polycythemia -Emotional changes

- Nausea -Emotional changes Rationale Nausea and vomiting may occur; this reflects a central emetic reaction to levodopa. Changes in affect, mood, and behavior are related to the toxic effects of carbidopa-levodopa. Insomnia, tremors, and agitation are side effects that may occur, not lethargy. Tachycardia and palpitations, not bradycardia, occur. Anemia and leukopenia, not polycythemia, are adverse reactions.

Which instruction about phenytoin will the nurse provide during discharge teaching to a client with epilepsy who is prescribed phenytoin for seizure control? -"Antiseizure medications will probably be continued for life." -"Phenytoin prevents any further occurrence of seizures." -"This medication needs to be taken during periods of emotional stress." -"Your antiseizure medication usually can be stopped after a year's absence of seizures."

-"Antiseizure medications will probably be continued for life." Rationale: Seizure disorders usually are associated with marked changes in the electrical activity of the cerebral cortex, requiring prolonged or lifelong therapy. Seizures may occur despite medication therapy; the dosage may need to be adjusted. A therapeutic blood level must be maintained through consistent administration of the medication irrespective of emotional stress. Absence of seizures will probably result from medication effectiveness rather than from correction of the pathophysiological condition.

Which response would a nurse give to a client who takes furosemide and digoxin and reports that everything looks yellow? -"This is related to your heart problems, not to the medication." -"I will hold the medication until I consult with your health care provider." -"It is a medication that is necessary, and that side effect is only temporary." -"Take this dose, and when I see your health care provider, I will ask about it."

-"I will hold the medication until I consult with your health care provider." Rationale The response "I will hold the medication until I consult with your health care provider" is a safe practice because yellow vision indicates digitalis toxicity. The response "This is related to your heart problems, not to the medication" is incorrect; yellow vision is not a symptom of heart disease. The response "It is a medication that is necessary, and that side effect is only temporary" is incorrect; yellow vision is not a temporary side effect. The response "Take this dose, and when I see your health care provider, I will ask about it" is unsafe.

The caregiver of a client with Alzheimer's disease asks the nurse for information about different treatment options that can help with memory or behavior problems. Which of the following responses by the nurse are correct? Select all that apply. -"Music therapy has been found to help some clients." -"Ginkgo biloba may help with memory." -"Acupuncture may be very relaxing." -"Donepezil (Aricept) may help slow cognitive decline." -"Garlic may help with this disease."

-"Music therapy has been found to help some clients." -"Ginkgo biloba may help with memory." -"Donepezil (Aricept) may help slow cognitive decline." Rationale: Some complementary and integrative health therapies may help with the symptoms of Alzheimer's disease. Music, art and dance therapies can help with behavior issues. Ginkgo biloba may be used to improve memory. Acupuncture may be a frightening experience for someone with Alzheimer's disease. Garlic is not a treatment for Alzheimer's disease. Donepezil (Aricept) is used to ease the symptoms associated with Alzheimer's disease.

A 65-year-old client is receiving amitriptyline. Which recommendation will the nurse make to the client concerning this medication? -'Obtain a complete cholesterol and lipid profile.' -'Have an eye examination to check for glaucoma.' -'Check your temperature daily for nighttime increases.' -'Watch for excessive sweating and possible weight loss.'

-'Have an eye examination to check for glaucoma.' Rationale: In addition to baseline laboratory tests, an older adult should have an eye examination with glaucoma testing when taking amitriptyline. It causes dilation of the pupil (mydriasis), which interferes with drainage of aqueous humor through the canal of Schlemm. Interfering with the outflow of aqueous humor will increase intraocular pressure and may cause a progressive loss of vision in clients with glaucoma. Amitriptyline does not affect cholesterol production or temperature regulation. It does not cause excessive sweating or weight loss, but it can increase appetite especially for sweets, resulting in weight gain.

935) The nurse provides medication discharge instructions to a client who received a prescription for digoxin. Which statement by the client leads the nurse to conclude that the teaching was effective? -'I will avoid foods high in potassium.' -'I must increase my intake of vitamin K.' -'I should adjust the dosage according to my activities.' -'It will be important to check my pulse rate daily.'

-'It will be important to check my pulse rate daily.' Rationale Checking the pulse rate daily is necessary for monitoring cardiac function; digoxin slows and strengthens the heart rate. Digoxin should be withheld, and the health care provider notified, if the pulse rate falls below a predetermined rate (e.g., 60 beats per minute). Hypokalemia increases the potential for digoxin toxicity; potassium intake may need to be increased, not decreased. An increase in the intake of foods rich in vitamin K is unnecessary; digoxin does not affect vitamin K or vitamin K clotting factors. Adjusting the dosage according to activities is not an appropriate decision for the client; the health care provider should make this decision.

A client with a seizure disorder will begin taking phenytoin. Which instructions will the nurse give to the client? -'Take the medication on an empty stomach.' -'Provide meticulous oral hygiene.' -'Taper off the medication if seizures are controlled for 3 months.' -'Stop taking the medication if you become pregnant.'

-'Provide meticulous oral hygiene.' Rationale: Phenytoin can cause gingival overgrowth that increases the risk for periodontal disease. The medication should be taken with food or milk to decrease gastrointestinal side effects. The health care provider should oversee any dosage adjustment or discontinuation. Although it can affect the developing fetus, current evidence suggests that the effect of seizure activity on the developing fetus may cause even greater problems.

Which instruction would the nurse give to clients prescribed psychotropic medications who are experiencing anticholinergic-like side effects? -'Restrict fluid intake.' -'Eat a diet high in carbohydrates.' -'Suck on sugar-free hard candies.' -'Avoid products that contain aspirin.'

-'Suck on sugar-free hard candies.' Rationale: Hard candy may produce salivation, which helps alleviate the anticholinergic-like side effect of dry mouth that is experienced with some psychotropics. Dry mouth increases the risk for cavities; candy with sugar adds to this risk. Fluids should be encouraged, not discouraged; fluids may alleviate dry mouth. Eating a diet high in carbohydrates and avoiding aspirin are unnecessary.

A client has been receiving digoxin. The client calls the clinic and complains of 'yellow vision.' Which response would the nurse provide? -'This is related to your illness rather than to your medication.' -'This is an expected side effect; you will become accustomed to it over time.' -'This side effect is only temporary. You should continue the medication.' -'The medication may need to be discontinued. Come to the clinic this afternoon.'

-'The medication may need to be discontinued. Come to the clinic this afternoon.' Rationale Yellow vision indicates digoxin toxicity; the medication should be withheld until the health care provider can assess the client and check the digoxin blood level. Yellow vision is related to digoxin therapy, not the client's underlying medical condition. Yellow vision is a sign of digoxin toxicity; taking more digoxin will escalate the digoxin toxicity.

The nurse teaches the parents of a child prescribed long-term phenytoin therapy about care. Which statement indicates the teaching has been effective? -'We give the medication between meals.' -'We'll call the clinic if her urine turns pink.' -'She's eating high-calorie foods, and we encourage fluids, too.' -'We'll have her massage her gums and floss her teeth frequently.'

-'We'll have her massage her gums and floss her teeth frequently.' Rationale: A common side effect of phenytoin is gingival hyperplasia. Meticulous oral hygiene may reduce the risk of this side effect. Phenytoin is strongly alkaline and should be administered with meals to help prevent gastric irritation. Pink urine may be observed during medication excretion; it is expected and does not require treatment. Avoidance of overeating and overhydration may result in better seizure control.

1104) Digoxin is prescribed for a client with heart failure. The nurse will assess for which signs and symptoms that indicate digoxin toxicity? Select all that apply. One, some, or all responses may be correct. -Nausea -Yellow vision -Irregular pulse -Increased urine output -Heart rate of 64 beats/minute

-*Nausea -*Yellow vision -*Irregular pulse Rationale Signs and symptoms of digoxin toxicity include bradycardia, headache, dizziness, confusion, nausea, and visual disturbances (blurred vision or yellow vision). In addition, electrocardiogram (ECG) findings may include heart block, atrial tachycardia with block, or ventricular dysrhythmias, all causing an irregular pulse. Increased urine output is an expected effect of improved cardiac output; a pulse rate of 64 beats/minute is an acceptable rate when a client is receiving digoxin.

Which assessment will the nurse conduct before administering digoxin to a client? -Apical heart rate -Radial pulse -Difference between carotid and radial pulses -Difference between apical and radial pulses

-Apical heart rate Rationale Because digoxin slows the heart rate, the apical pulse should be counted for 1 minute before administration. If the apical rate is below a preset parameter (usually 60 beats/minute), digoxin should be withheld because its administration may further decrease the heart rate. Some protocols permit waiting for 1 hour and retaking the apical rate; the result determines if it is administered or if the health care provider is notified. Obtaining the radial pulse on the left side is not as accurate as an apical pulse; the client also may have an atrial dysrhythmia, which cannot be detected through a radial rate alone. Obtaining the radial pulse in both right and left arms is not as accurate as an apical pulse; the client also may have an atrial dysrhythmia, which cannot be detected through a radial rate alone. Obtaining the difference between apical and radial pulses is a pulse deficit, not a pulse rate.

A client suspected of having myasthenia gravis is scheduled for an edrophonium chloride test. To treat a common complication associated with the test, the nurse will have which medication available? -Atropine -Phenytoin -Neostigmine -Diphenhydramine

-Atropine Rationale: Atropine, an anticholinergic, should always be available to treat a cholinergic crisis (sudden, severe episode of muscle weakness that affects breathing and swallowing) should the edrophonium chloride test trigger this response. Phenytoin is an anticonvulsant that will not avert or treat complications resulting from an edrophonium test. Neostigmine is a cholinergic that has the same action as edrophonium chloride; it is contraindicated if a cholinergic crisis occurs. Diphenhydramine is an antihistamine that will not avert complications or effectively treat a cholinergic crisis.

During a procedure, the client's heart rate drops to 38 beats/min. Which medication is indicated to treat bradycardia? -Digoxin -Lidocaine -Amiodarone -Atropine sulfate

-Atropine sulfate Rationale Atropine blocks vagal stimulation of the sinoatrial (SA) node, resulting in an increased heart rate. Digoxin slows the heart rate; hence, it would not be indicated in this situation. Lidocaine decreases myocardial sensitivity and will not increase the heart rate. Amiodarone is an antidysrhythmic medication used for ventricular tachycardia; it will not stimulate the heart rate.

A client develops extrapyramidal effects after taking a neuroleptic medication, and the nurse notes extrapyramidal effects. Which medication can limit these side effects? -Zolpidem -Hydroxyzine -Dantrolene -Benztropine mesylate

-Benztropine mesylate Rationale: Benztropine mesylate, an anticholinergic, helps balance neurotransmitter activity in the central nervous system (CNS) and helps control extrapyramidal tract symptoms. Zolpidem is a sedative-hypnotic medication used for short-term insomnia. Hydroxyzine is a sedative that depresses activity in the subcortical areas in the CNS; it is used to reduce anxiety. Dantrolene, a muscle relaxant, has a direct effect on skeletal muscle by acting on the excitation-contraction coupling of muscle fibers and not at the level of the CNS, unlike most other muscle relaxation medications.

A client with Parkinson disease is admitted to the hospital. Which medication is prescribed to improve the physical manifestations of Parkinson disease? -Carbidopa-levodopa -Isocarboxazid -Dopamine -Pyridoxine (vitamin B 6)

-Carbidopa-levodopa Rationale Levodopa crosses the blood-brain barrier and converts to dopamine, a substance depleted in Parkinson disease. Isocarboxazid is a monoamine oxidase inhibitor used for the treatment of psychological symptoms associated with severe depression, not physiological symptoms of Parkinson disease. Dopamine is not prescribed for this purpose because it does not cross the blood-brain barrier. Pyridoxine can reverse the effects of some antiparkinsonian medications and is contraindicated.

The client with a seizure disorder receives intravenous (IV) phenytoin. The nurse will monitor closely for which condition? -Cardiac dysrhythmias -Hypoglycemia -Polycythemia -Paradoxical excitation

-Cardiac dysrhythmias Rationale IV phenytoin was once used to treat dysrhythmias until better medications were developed. It depresses both atrial and ventricular conduction, and so it can cause significant dysrhythmias. It can also cause hyperglycemia (not hypoglycemia) and pancytopenia (not polycythemia). Paradoxical excitation is not a known issue; it has a depressant effect resulting in drowsiness.

The nurse teaches a client's family about the administration of donepezil for treatment of dementia of the Alzheimer type. Which side effect identified by the caregiver indicates to the nurse that further teaching is needed? -Nausea -Dizziness -Headache -Constipation

-Constipation Rationale: Donepezil, a cholinesterase inhibitor, may cause nausea, vomiting, increased salivation, diarrhea, and involuntary defecation related to the increase in gastrointestinal secretions and activity caused by parasympathetic nervous stimulation; it does not cause constipation. Common side effects of donepezil include anorexia, nausea, and vomiting that result from stimulation of the parasympathetic nervous system. Dizziness and headache are common side effects of donepezil that result from central nervous system cholinergic effects.

A client with myasthenia gravis is receiving pyridostigmine bromide to control symptoms. Recently, the client has begun experiencing increased difficulty in swallowing. Which nursing action is effective in preventing aspiration of food? -Place a tracheostomy set in the client's room. -Assess respiratory status after meals. -Request for the diet to be changed from soft to clear liquids. -Coordinate mealtimes with the peak effect of the medication.

-Coordinate mealtimes with the peak effect of the medication. Rationale Dysphagia should be minimized during peak effect of pyridostigmine bromide, thereby decreasing the probability of aspiration. A tracheostomy set is a treatment for, rather than equipment to prevent, aspiration. Although it is vital that the client's respiratory function be monitored, assessing the client's respiratory status will not prevent aspiration. There are insufficient data to determine whether changing the diet from soft foods to clear liquids is appropriate; also, liquids are aspirated more easily than semisolids.

A client is treated with lorazepam for status epilepticus. Which effect of lorazepam is the reason it is given? -Decreases anxiety associated with seizures -Promotes rest after the seizure episode -Depresses the central nervous system (CNS) -Provides amnesia for the convulsive episode

-Depresses the central nervous system (CNS) Rationale: Lorazepam is used to treat status epilepticus because it depresses the CNS. It also functions as an anxiolytic and sedative and can cause anterograde amnesia; however, these are not the reasons it is prescribed for status epilepticus.

564) An infant with congenital heart disease is prescribed digoxin and furosemide upon discharge. Which sign would the nurse instruct the parents to be alert for? -Difficulty feeding with vomiting -Cyanosis during periods of crying -Daily naps lasting more than 3 hours -A pulse rate faster than 100 beats/min

-Difficulty feeding with vomiting Rationale Vomiting and feeding issues are early signs of digoxin toxicity. Cyanosis is expected in a crying infant with heart disease because the energy expenditure exceeds the body's ability to meet the oxygen demand. Long naps are expected; infants routinely require several naps, and an infant with heart disease requires long rest periods. The pulse rate of an infant receiving digoxin should remain faster than 100 beats/min

Which medication would the nurse expect to administer to actively reverse the overdose sedative effects of benzodiazepines? -Lithium -Flumazenil -Methadone -Chlorpromazine

-Flumazenil Rationale Flumazenil is the medication of choice in the management of overdose when a benzodiazepine is the only agent ingested by a client not at risk for seizure activity. Flumazenil competitively inhibits activity at benzodiazepine recognition sites on gamma-aminobutyric acid-benzodiazepine receptor complexes. Lithium is used in the treatment of mood disorders. Methadone is used for narcotic addiction withdrawal. Chlorpromazine is contraindicated in the presence of central nervous system depressants.

Which medication acts as an antidote to benzodiazepine? -Zolpidem -Temazepam -Suvorexant -Flumazenil

-Flumazenil Rationale: Flumazenil is a benzodiazepine receptor antagonist and is used to reverse conscious sedation induced by benzodiazepines. Zolpidem is a nonbenzodiapepine, sedative used to treat insomnia. Temazepam and is a benzodiazepine. Suvorexant is in a class of medications called orexin receptor antagonists that works by blocking the action of a certain natural substance in the brain that causes wakefulness.

Which nursing action is appropriate when administering imipramine? -Telling the client steroids will not be prescribed -Warning the client not to eat cheese -Monitoring the client for increased tolerance -Having the client checked for increased intraocular pressure

-Having the client checked for increased intraocular pressure Rationale: Glaucoma is one of the side effects of imipramine, and the client should be taught about the symptoms. The prescribing of steroids and avoiding cheese are true of monoamine oxidase inhibitors (MAOIs); imipramine is not an MAOI. Tolerance is not an issue with tricyclic antidepressants such as imipramine.

Pyridostigmine bromide is prescribed for a client with myasthenia gravis. The nurse evaluates that the medication regimen is understood when the client makes which statement? -'I will take the medication on an empty stomach.' -I need to set an alarm so I take the medication on time.' -'It will be important to check my heart rate before taking the medication.' '-I should monitor for an increase in blood pressure after taking the medication.'

-I need to set an alarm so I take the medication on time.' Rationale: Pyridostigmine is a vital medication that must be taken on time; a missed or late dose can result in severe respiratory and neuromuscular consequences or even death. Pyridostigmine should be taken with a small amount of food to prevent gastric irritation. It is unnecessary to take the pulse rate before taking pyridostigmine. Pyridostigmine may cause hypotension, not hypertension, which is a sign of cholinergic crisis.

An 11-year-old client reports having bedwetting issues (enuresis). Which medication would the nurse anticipate when developing a teaching plan ? -Alprazolam -Imipramine -Lithium salts -Clomipramine

-Imipramine Rationale: Certain conditions of pediatric clients necessitate the usage of tricyclic antidepressant medications as an adjuvant. Childhood enuresis is one such condition that necessitates the administration of imipramine. Alprazolam is the medication of choice for treating anxiety disorders. Lithium salt is prescribed to treat bipolar disorders. Clomipramine is a tricyclic antidepressant medication prescribed for treating obsessive-compulsive disorder.

Digoxin is prescribed for a client. Which therapeutic effect of digoxin would the nurse expect? -Decreased cardiac output -Decreased stroke volume of the heart -Increased contractile force of the myocardium -Increased electrical conduction through the atrioventricular (AV) node

-Increased contractile force of the myocardium Rationale Digoxin produces a positive inotropic effect that increases the strength of myocardial contractions and thus cardiac output. The positive inotropic effect of digoxin increases, not decreases, cardiac output. Digoxin increases the strength of myocardial contractions (positive inotropic effect) and slows the heart rate (negative chronotropic effect); these effects increase the stroke volume of the heart. Digoxin decreases the refractory period of the AV node and decreases conduction through the sinoatrial (SA) and AV nodes.

A client presents to the clinic for a follow-up appointment after starting pyridostigmine for management of myasthenia gravis. Which new client problems are adverse effects of pyridostigmine? Select all that apply. One, some, or all responses may be correct. -Respiratory depression -Increased urinary frequency -Diplopia -Muscle twitching -Diarrhea

-Increased urinary frequency -Muscle twitching -Diarrhea Rationale: Pyridostigmine is an acetylcholinesterase inhibitor. By inhibiting the enzyme that breaks down acetylcholine, it increases cholinergic activity. The increased cholinergic activity is responsible for the most common adverse effects of pyridostigmine such as diarrhea and increased urinary frequency. Another adverse effect is muscle twitching that occurs as pyridostigmine affects the muscle weakness that is characteristic of myasthenia gravis. Respiratory depression is a clinical manifestation of myasthenia gravis that can occur due to respiratory muscle weakness. Weakness of eye muscles results in ptosis or diplopia in about half of clients with myasthenia gravis.

The nurse provides care for a client with a long history of alcohol abuse. Which medication would the nurse anticipate will be prescribed for the client to prevent symptoms of withdrawal? -Lorazepam -Phenobarbital -Chlorpromazine -Disulfiram

-Lorazepam Rationale: Lorazepam is most effective in preventing the signs and symptoms associated with withdrawal from alcohol. It depresses the central nervous system by potentiating gamma-aminobutyric acid, an inhibitory neurotransmitter. Phenobarbital is used to prevent withdrawal symptoms associated with barbiturate use. Chlorpromazine, an antipsychotic medication, is not used for alcohol withdrawal. Disulfiram does not prevent symptoms; it is aversion therapy that causes symptoms when alcohol is ingested.

A client takes furosemide and digoxin for heart failure. Why would the nurse advise the client to drink a glass of orange juice every day? -Maintaining potassium levels -Preventing increased sodium levels -Limiting the medications' synergistic effects -Correcting the associated dehydration

-Maintaining potassium levels Rationale Orange juice is an excellent source of potassium. Furosemide promotes excretion of potassium, which can result in hypokalemia. Digoxin toxicity can occur in the presence of hypokalemia. Neither medication increases sodium levels. Digoxin does not potentiate the action of furosemide; therefore the client should not experience dehydration. Orange juice will not prevent an interaction between digoxin and furosemide.

Which primary reason identifies why oxazepam is given during detoxification? -Prevents injury when seizures occur -Enables the client to sleep better during periods of agitation -Encourages the client to accept treatment for alcoholism -Minimizes withdrawal symptoms the client may experience

-Minimizes withdrawal symptoms the client may experience Rationale Oxazepam potentiates the actions of gamma-aminobutyric acid, especially in the limbic system and reticular formation, and thus it minimizes withdrawal symptoms. This medication helps reduce the risk for seizures but does not prevent injury during a seizure. Enabling the client to sleep better during periods of agitation is not the purpose of the medication. The ability of the client to accept treatment depends on the person's readiness to accept the reality of the problem.

When a client with type 1 diabetes develops heart failure, digoxin is prescribed. Which nursing action is important to include when planning care? -Administer the digoxin 1 hour after the client's morning insulin. -Monitor the client for cardiac dysrhythmias. -Monitor for increased risk of hyperglycemia. -Increase digoxin dosage if insulin requirements are increased.

-Monitor the client for cardiac dysrhythmias. Rationale The speed of conduction is decreased when digoxin is given, and this can result in a variety of cardiac dysrhythmias. The risk for hyperglycemia is not increased. Administration times for insulin and digoxin do not have to be coordinated. Dosage of digoxin is not dependent on insulin dosage.

Which nursing intervention is important when caring for clients receiving intravenous (IV) digoxin? Select all that apply. One, some, or all responses may be correct. -Monitor the heart rate closely. -Check the blood levels of digoxin. -Administer the dose over 1 minute. -Monitor the serum potassium level. -Give the medication with other infusing medications.

-Monitor the heart rate closely. -Check the blood levels of digoxin. -Monitor the serum potassium level. Rationale Bradycardia or other dysrhythmias may occur; therefore the heart rate and rhythm should be monitored. Electrocardiogram (ECG) monitoring should be continuous. The digoxin level is checked before administration to avoid toxicity. A low serum potassium level when digoxin is administered can contribute to toxicity. Digoxin should be given over a 5-minute period through a Y-tube or three-way stopcock. There are many syringe, Y-site, and additive incompatibilities; the manufacturer recommends that digoxin not be administered with other medications.

Which instruction would the nurse provide to parents of a school-age child who has been on long-term phenytoin therapy to prevent side effects? -Provide good oral hygiene. -Administer the medication between meals. -Watch for a reddish-brown discoloration of urine. -Supplement the diet with high-calorie foods.

-Provide good oral hygiene. Rationale: These procedures reduce the risk for gingival hyperplasia, a side effect of phenytoin. This medication is strongly alkaline and should be administered with meals to help prevent gastric irritation. Discoloration of the urine may occur during medication excretion; it does not cause physiological problems. Avoiding overeating and overhydration may result in better seizure control.

Diazepam is administered to the client with status epilepticus. In addition to decreasing central neuronal activity, which effect would the nurse anticipate? -Relaxing of peripheral muscles -Decreased heart rate -Dilation of airways -Hypertension

-Relaxing of peripheral muscles Rationale: Diazepam is a benzodiazepine indicated for treatment of anxiety, muscle spasms, and seizures. Peripheral muscles may relax as a result of the antispasmodic effects. Diazepam does not slow the heart rate. Diazepam does not dilate the bronchial airways and may cause bronchoconstriction, though this is uncommon. Diazepam may cause hypotension, not hypertension.

The nurse administers carbidopa-levodopa to a client with Parkinson's disease. Which activity describes the mechanism of action of this medication? -Increase in acetylcholine production -Regeneration of injured thalamic cells -Improvement in myelination of neurons -Replacement of a neurotransmitter in the brain

-Replacement of a neurotransmitter in the brain Rationale Carbidopa-levodopa is used because levodopa is the precursor of dopamine. It is converted to dopamine in the brain cells, where it is stored until needed by axon terminals; it functions as a neurotransmitter. Regeneration of injured thalamic cells is not an action of this medication; neurons do not regenerate. Increase in acetylcholine production and improvement in myelination of neurons are not actions of this medication.

922) A client is given a loading dose of digoxin and placed on a maintenance dose of digoxin 0.25 mg by mouth daily. Which responses would the nurse expect the client to exhibit when a therapeutic effect of digoxin is achieved? -Resolution of heart failure -Decreased anginal episodes -Conversion of atrial fibrillation -Decreased blood pressure

-Resolution of heart failure Rationale Digoxin improves cardiac output to improve heart failure. Digoxin is not an antianginal medication; if it decreases angina as a result of controlling heart failure, it is a secondary effect. Digoxin may be given to control a rapid ventricular response to atrial fibrillation, but it does not convert the rhythm. Digoxin has a negligible effect on blood pressure; therefore it is not an antihypertensive medication.

Which mechanism of action would the nurse identify for levodopa therapy prescribed to a client diagnosed with Parkinson disease? -Blocks the effects of acetylcholine -Increases the production of dopamine -Restores the dopamine levels in the brain -Promotes the production of acetylcholine

-Restores the dopamine levels in the brain Rationale Levodopa is a precursor of dopamine, a catecholamine neurotransmitter; it increases dopamine levels in the brain that are depleted in Parkinson disease. Blocking the effects of acetylcholine is accomplished by anticholinergic medications. Increasing the production of dopamine is ineffective because it is believed that the cells that produce dopamine have degenerated in Parkinson disease. Levodopa does not affect acetylcholine production.

The nurse is providing discharge medication teaching to a client who will be taking furosemide and digoxin after discharge from the hospital. Which information is important for the nurse to include in the teaching plan? -Maintenance of a low-potassium diet -Avoidance of foods high in cholesterol -Signs and symptoms of digoxin toxicity -Importance of monitoring output

-Signs and symptoms of digoxin toxicity Rationale The risk of digoxin toxicity increases when the client is receiving digoxin and furosemide, a loop diuretic; loop diuretics can cause hypokalemia, which potentiates the effects of digoxin, leading to toxicity. Digoxin toxicity can result in dysrhythmias and death. When a client is receiving a loop diuretic, the diet should be high in potassium. Although teaching the need to avoid foods high in cholesterol may be included in the teaching plan, it is not the priority. It is not necessary to monitor output.

Pyridostigmine is prescribed for a client with myasthenia gravis. Why would the nurse instruct the client to take pyridostigmine about 1 hour before meals? -This timing limits first pass metabolism. -Taking it on an empty stomach increases absorption. -Taking it before meals decreases gastric irritation. -Taking it before meals improves the ability to chew.

-Taking it before meals improves the ability to chew. Rationale: Peak action of the medication will occur during meals to promote chewing and swallowing and prevent aspiration. It should be given with a small amount of food to prevent gastric irritation. First pass metabolism is a process of metabolism, which is not affected by medication timing. Absorption is not affected significantly by the presence of food in the stomach. Gastric irritation is reduced best by the administration of medications with food, not on an empty stomach.

A client with myasthenia gravis begins taking pyridostigmine. Two days later, the client develops loose stools and increased salivation. Which conclusion would the nurse make about these new developments? -The client is experiencing a myasthenic crisis. -The medication is causing cholinergic side effects. -The medication is triggering a paradoxical reaction. -The client is exhibiting toxic effects of the medication.

-The medication is causing cholinergic side effects. Rationale: Because this medication inhibits the destruction of acetylcholine, parasympathetic activity may increase, resulting in cholinergic side effects such as diarrhea and increased salivation. The signs do not indicate a myasthenic crisis. Myasthenic crisis is characterized by difficulty breathing or speaking, morning headaches, feeling tired during the daytime, waking up frequently at night, not sleeping well, a weak cough with increased secretions (mucus or saliva), an inability to clear secretions, a weak tongue, trouble swallowing or chewing, and weight loss. Side effects are not temporary and not paradoxical; they continue as long as the medication is continued. The dosage may be adjusted or an anticholinergic may be given to limit side effects. Toxicity or cholinergic crisis is manifested by increased muscle weakness, including muscles of respiration.

Which would the nurse include when teaching a client with Parkinson disease about carbidopa-levodopa? -Multivitamins should be taken daily. -A high-protein diet should be followed. -The medication should be taken with meals. -Alcohol consumption should be in moderation.

-The medication should be taken with meals. Rationale Carbidopa-levodopa should be taken with meals to reduce the nausea and vomiting that commonly are caused by this medication. Multivitamins are contraindicated; vitamins may contain pyridoxine (vitamin B 6), which diminishes the effects of levodopa. A high-protein diet is contraindicated. Sinemet contains levodopa, an amino acid that may increase blood urea nitrogen levels. Also, some proteins contain pyridoxine, which increases the peripheral metabolism of levodopa, decreasing the amount of levodopa crossing the blood-brain barrier.

Status epilepticus develops in an adolescent with a seizure disorder who is taking antiseizure medication. Which reason would the nurse identify as the most common reason for the development of status epilepticus? -The provider failed to account for a growth spurt. -The amount prescribed is insufficient to cover activities. -The prescribed antiseizure medication probably is not taken consistently. -The client is prescribed a medication that is ineffective in preventing seizures.

-The prescribed antiseizure medication probably is not taken consistently. Rationale: Skipping doses of the medication is a form of denial that an adolescent client may engage in once the seizures are controlled; also, adolescents tend to feel invincible. The dosage is based not on activity but on the type of seizure. Medications are prescribed according to the type of seizure and are effective if taken consistently. The dosage of antiepileptic medications is based on many factors, including age, type of medication, and presence of infection, as well as on changes in height and weight.

In addition to hydration, parenteral lorazepam is prescribed for a client during alcohol withdrawal delirium. Which primary purpose accurately explains why this medication is given during detoxification? -To prevent injury when seizures occur -To enable the client to sleep better during periods of agitation -To reduce the anxiety tremor state and prevent more serious withdrawal symptoms -To calm the client and promote acceptance of the treatment plan

-To reduce the anxiety tremor state and prevent more serious withdrawal symptoms Rationale: Lorazepam potentiates the actions of gamma-aminobutyric acid, which reduces anxiety and irritability that are common during withdrawal. This medication helps reduce the risk of seizures but does not prevent physical injury if a seizure occurs. Although the medication may enable the client to sleep better during periods of agitation, this is not its primary objective. The ability of the client to accept treatment depends on readiness to accept the reality of the problem.

Which medication class includes amitriptyline? -Tricyclics -Monoamine oxidase inhibitors (MAOIs) -Selective serotonin reuptake inhibitors (SSRIs) -Serotonin-norepinephrine reuptake inhibitors (SNRIs)

-Tricyclics Rationale: Amitriptyline is one of several tricyclic antidepressants used to treat anxiety disorders. It is not an MAOI, SSRI, or SNRI.

Which adverse effect of imipramine requires further assessment and possible immediate medical intervention? -Dry mouth -Weight gain -Blurred vision -Urinary hesitancy

-Urinary hesitancy Rationale: Urinary hesitancy and retention are adverse effects of imipramine that may require immediate medical intervention. Dry mouth, weight gain related to increased appetite, and blurred vision may occur as side effects of imipramine; they usually decrease over time or can be managed through nursing interventions.

1486) The nurse is reviewing medication instructions with parents of an infant receiving digoxin and spironolactone. Which parental response indicates instructions have been understood? -Activity should be restricted. -Orange juice should be given daily. -Vomiting should be reported to the health care provider. -Anti-inflammatory medications should be avoided.

-Vomiting should be reported to the health care provider. Rationale Vomiting is a classic sign of digoxin toxicity, and the health care provider must be notified. Infants regulate their own activity according to their energy level. Orange juice is rarely needed because spironolactone spares potassium. There is no restriction on anti-inflammatory medications with spironolactone.

567) According to developmental norms for a 5-year-old child, the nurse would hold digoxin if an apical heart rate falls below which number? 70 beats/min 80 beats/min 90 beats/min 100 beats/min

70 beats/min Rationale The purpose of digoxin is to slow and strengthen the apical rate. The apical rate for a healthy child of 5 years is 70 to 110 beats/min. If the apical rate is slow, administration of the medication may lower the apical rate to an unsafe level.

To which nursing home resident could a nurse safely administer tricyclic antidepressants without questioning the health care provider's order? A client with mild hypertension BA client with narrow-angle glaucoma CA client with coronary artery disease (CAD) DA client with benign prostatic hypertrophy (BPH)

A client with mild hypertension Rationale: Tricyclics can be safely administered to the hypertensive client. The expected anticholinergic effects of tricyclic antidepressants include difficulty in urination, which is why TCAs are contraindicated with BPH. TCAs are also contraindicated in narrow-angle glaucoma (they can cause elevated pressure in the eyes) and for certain heart abnormalities.

The nurse is providing care for a client admitted to the hospital with a diagnosis of digoxin toxicity. The client reports more than usual urine output over the previous 48 hours, because of the prescribed diuretic. Which assessment finding does the nurse anticipate? AMuscle weakness or cramping BBlood in the urine CHypertension DTinnitus

AMuscle weakness or cramping Rationale: The client with heart failure on digoxin and a diuretic is at risk for hypokalemia. The digoxin binds to the potassium receptor of the sodium/potassium ATPase pump. The increased urine output makes hypokalemia likely and thus it is more likely for digoxin toxicity to occur. Symptoms of hypokalemia include muscle weakness and cramping. The digoxin toxicity will not cause blood in the urine, or tinnitus or hypertension.

An 80-year-old client who is taking digoxin reports nausea, vomiting, abdominal cramps and halo vision. Which laboratory result should the nurse evaluate first? APotassium levels BBlood pH CMagnesium levels DBlood urea nitrogen

APotassium levels Rationale: Nausea, vomiting, abdominal cramps and halo vision are classic signs of digitalis toxicity. The most common cause of digitalis toxicity is a low potassium level. Clients are to be taught that it is important to have adequate potassium intake, especially if taking loop or thiazide diuretics that enhance the loss of potassium.

A nurse is educating a client about digoxin toxicity. Which statement made by the client indicates that more teaching is needed? A"High levels of digoxin can cause vision changes." B"I must report a strong pulse of 62 beats per minute to the health care provider." C"I should report nausea and vomiting lasting more than a few days." D"I will let the health care provider know if my pulse feels uneven or misses beats."

B"I must report a strong pulse of 62 beats per minute to the health care provider." Rationale: Digoxin is used to increase the strength of heart contraction. The expected effect of digoxin use is a slower, strong pulse. The client should be instructed to check their pulse prior to taking this medication and to note the rate and if the rhythm is irregular. If the heart rate is less than 60 or greater than 100 the client should not take the medication. Therefore, a strong pulse of 62 is a therapeutic effect of this medication and would not warrant a call to health care provider. This needs to be clarified with the client. The other statements indicate understanding of the medication. Digoxin toxicity would cause irregular pulse, loss of appetite, nausea, vomiting and vision changes. The client should be alert to these clinical manifestations and call the health care provider if they experience any of these changes.

The nurse has administered fentanyl, atropine, cefazolin and benzocaine to a client for an endoscopic procedure. The nurse is monitoring the client and notes that the heart rate has increased from the pre-procedure baseline. The nurse knows that which of the following medications is most likely responsible for the client's increased heart rate? AFentanyl BAtropine CCefazolin DBenzocaine

BAtropine Rationale: Procedural sedation is used in endoscopic procedures as an effective way to provide an appropriate degree of pain and anxiety control; memory loss; and decreased awareness. The most commonly used medication regimen for gastrointestinal endoscopic procedure is still the combination of benzodiazepines, opioids, anticholinergics and topical anesthetics. Atropine is an anticholinergic drug that is used to dry secretions during the procedure. However, it can also increase the heart rate and dilate the pupils and is the most likely cause for the increased heart rate. Fentanyl is an opioid analgesic and short-term central nervous system (CNS) depressant and tends to slow breathing and lower heart rate and blood pressure. Benzocaine is a topical anesthetic and cefazolin is an antibiotic; neither should affect the heart rate.

The nurse is caring for a client with Parkinson's disease. Which finding indicates that the client might be experiencing an adverse side effect from the dopamine-enhancing drugs? AUrinary retention BHallucinations CKidney failure DHypertensive urgency

BHallucinations Rationale: Carbidopa-levodopa-entacapone is the treatment of choice for clients with Parkinson's disease. Common side effects include dyskinesia, confusion and dizziness. Serious side effects include hallucinations, paranoia and agitation. Hallucinations may be relieved by decreasing the dose of levodopa, but this may decrease the effect of the drug on the motor symptoms of Parkinson's disease.

A hospitalized 8-month-old infant is receiving digoxin to treat Tetralogy of Fallot. Prior to administering the next dose of the medication, the parent reports that the baby vomited one time, just after breakfast. The infant's heart rate is 92 bpm. What action should the nurse take? AGive the scheduled dose after the client is done eating lunch. BHold the medication and notify the primary health care provider. CReduce the next dose by half and then resume the normal medication schedule. DDouble the next dose to make up for the medication lost from vomiting.

BHold the medication and notify the primary health care provider. Rationale: Toxic side effects of digoxin include bradycardia, dysrhythmia, nausea, vomiting, anorexia, dizziness, headache, weakness and fatigue. It isn't typically necessary to hold the medication for infants and children if there is only one episode of vomiting. However, it is appropriate to hold the medication and notify the primary health care provider (HCP) of the vomiting episode and the lower than normal heart rate. A digoxin level may need to be drawn. The normal resting heart rate for infants 1 to 11 months old is 100 to 160 bpm.

The client diagnosed with heart failure is prescribed oral digoxin. What is the priority nursing assessment for this medication? AMonitor serum electrolytes and creatinine BMeasure apical pulse prior to administration CMaintain accurate intake and output ratios DMonitor blood pressure every 4 hours

BMeasure apical pulse prior to administration Rationale: Digoxin is an antiarrhythmic and an inotropic drug. It works to increase cardiac output and slow the heart rate. The priority assessment is to measure the apical pulse for one minute prior to administering the drug. The nurse will withhold the dose and notify the healthcare provider if the apical rate is less than 60 beats per minute. Intake and output ratios and daily weights should be monitored for a client in heart failure, but this is not the priority assessment. Impaired renal function may contribute to drug toxicity, which is why the nurse will monitor serum electrolytes, creatinine and BUN; the nurse should also monitor serum digoxin levels.

The nurse is reviewing medication instructions with a client who is taking digoxin. The nurse should reinforce to the client to report which of the following side effects? ARash, dyspnea, edema BNausea, vomiting, fatigue CHunger, dizziness, diaphoresis DPolyuria, thirst, dry skin

BNausea, vomiting, fatigue Rationale: Digoxin is considered an antidysrhythmic and inotrope, that is used to treat atrial dysrhythmias and congestive heart failure. The medication produces a positive inotropic effect, prolongs the refractory period and slows conduction through the sinoatrial (SA) and atrioventricular (AV) nodes. Overall, digoxin increases cardiac output and slows the heart rate. The effects of digoxin produce many side effects and clients who take digoxin are at risk for digoxin toxicity. Because digoxin improves cardiac output, side effects of the medication would not include dyspnea or edema. Rashes are also not considered a side effect of digoxin. Common manifestations of digoxin toxicity include nausea, vomiting and fatigue. Hunger, dizziness and diaphoresis, together, are not considered side effects of digoxin. Although dizziness could occur with another side effect of digoxin, such as bradycardia. Polyuria, thirst and dry skin are not considered side effects of digoxin.

The nurse is preparing to administer digoxin to a client with recurring atrial fibrillation. Which laboratory value should be of highest concern for the nurse? AHemoglobin 9.4 g/dL BSerum potassium 3.1 mEq/L CSerum creatinine 1.9 mg/dL DB-type natriuretic peptide 140 pg/mL

BSerum potassium 3.1 mEq/L Rationale: Digoxin is a cardiac glycoside used to treat atrial dysrhythmias and heart failure. Because digoxin competes with potassium ions, digoxin should not be given when the client's potassium level is below normal range. Giving digoxin to a client with hypokalemia can cause digoxin toxicity and life-threatening cardiac dysrhythmias. Although all of the lab values are outside of normal range, the low potassium level (normal range 3.5-5.0 mEq/L) should be of highest concern for the client at this time. The nurse should hold the digoxin and notify the health care provider.

The nurse is educating a client with seizure disorder about newly prescribed phenytoin. Which statement should the nurse include in the teaching? ABlood work will be required if you have a seizure while taking this medication BYou will need to have routine visits with a dentist when taking this medication CIt is normal to have a change in your gait when you first start this medication DAvoid grapefruit juice when taking this medication

BYou will need to have routine visits with a dentist when taking this medication Rationale: Phenytoin is the first-line medication for the treatment of seizures. Clients should be instructed that they will need routine lab work to ensure that they are at a therapeutic level with the medication, even if they have been seizure-free. This medication can cause gingival hyperplasia, which will require routine dental visits. The client does not need to avoid grapefruit juice with this medication. Difficulty with hand and gait coordination could indicate toxicity and should be reported to the healthcare provider.

A client with myasthenia gravis has been receiving neostigmine and asks about its action. Which information would the nurse consider when formulating a response? - Stimulates the cerebral cortex - Blocks the action of cholinesterase - Replaces deficient neurotransmitters - Accelerates transmission along neural sheaths

Blocks the action of cholinesterase Rationale: Neostigmine, an anticholinesterase, inhibits the breakdown of acetylcholine, thus prolonging neurotransmission. Neostigmine's action is at the myoneural junction, not the cerebral cortex. Neostigmine prevents neurotransmitter breakdown, but it is not a neurotransmitter. Neostigmine's action is at the myoneural junction, not the sheath.

A client with hypertensive heart disease who had an acute episode of heart failure is to be discharged on a regimen of metoprolol and digoxin. Which outcome would the nurse anticipate when metoprolol is administered with digoxin? Headaches Bradycardia Hypertension Junctional tachycardia

Bradycardia Rationale Metoprolol and digoxin both exert a negative chronotropic effect, resulting in a decreased heart rate. Metoprolol reduces, not produces, headaches. These medications may cause hypotension, not hypertension. These medications may depress nodal conduction; therefore junctional tachycardia would be less likely to occur.

The daughter of a client with Alzheimer's disease asks the nurse, "Will the medication my mother is taking cure her dementia?" What is the best response by the nurse? A"It will help your mother live independently again." B"It is used to halt the progression of Alzheimer's disease." C"It will not improve dementia but can help control emotional responses." D"It will provide a steady improvement in memory."

C"It will not improve dementia but can help control emotional responses." Rationale: Drug therapy for Alzheimer's disease such as memantine and donepezil produce modest improvements in cognition, behavior, and function, and slightly delayed disease progression. They do not reverse the dementia or halt the progression of Alzheimer's disease. At best, drugs currently in use may slow loss of memory and improve cognitive functions (e.g., memory, thought, reasoning) and emotional lability. However, these improvements are modest and last a short time and for many clients, even these modest goals are elusive.

A client is prescribed digoxin 0.25 mg by mouth daily. The health care provider has written a new order to give metoprolol tartrate 25 mg twice a day by mouth. In assessing the client prior to administering the medications, which finding should the nurse report to the health care provider? AUrine output of 50 mL/hour BRespiratory rate of 16 CBlood pressure of 94/60 DHeart rate of 76 BPM

CBlood pressure of 94/60 Rationale: Both medications decrease the heart rate. Metoprolol (Lopressor)affects blood pressure. Therefore, the heart rate and blood pressure must be within normal range (HR 60 to 100 BPM and systolic BP greater than 100 mm Hg) in order to safely administer both medications.

The nurse is monitoring a 4-month-old infant who is prescribed digoxin. The infant's blood pressure is 92/78 mm Hg; resting pulse is 78 beats per minute; respirations are 28 breaths per minute; and serum potassium level is 4.8 mEq/L. The infant is irritable and has vomited twice since receiving the morning dose of digoxin. Which finding is most indicative of digoxin toxicity? AIrritability BVomiting CBradycardia DDyspnea

CBradycardia Rationale: The most common sign of digoxin toxicity in children is bradycardia which is a heart rate below 100 beats per minute in an infant. Normal resting heart rate for infants 1-11 months-old is 100-160 beats per minute.

Question 16 A nurse is preparing to administer morning medications to a client with heart failure. The morning lab values are: sodium 142 mEq/L (142 mmol/L), potassium 2.9 mEq/L (2.9 mmol/L), digoxin level 1.4 ng/mL. Which of the following medications should the nurse not administer until after speaking with the health care provider? ASpironolactone BCarvedilol (Coreg) CDigoxin (Lanoxin) DFerrous sulfate

CDigoxin (Lanoxin) Rationale: Because the potassium levels are low (normal is 3.5 to 5 mEq/L or 3.5 to 5 mmol/L), the nurse should not give the digoxin; hypokalemia can predispose a person to digoxin toxicity. The other medications can be administered. Although carvedilol can increase plasma digoxin concentration, the digoxin level is normal. Spironolactone is a potassium-sparing diuretic and because the potassium level is low, this too can be given. Ferrous sulfate does not affect the given lab values.

A nurse is teaching parents of a child recently prescribed the medication phenytoin for seizure control. Which side effect will the nurse include? AHypertension BInsomnia CGingival hyperplasia DIncreased appetite

CGingival hyperplasia Rationale: Gingival hyperplasia (overgrowth of the gums) is a common side effect of phenytoin. Other common side effects include ataxia, central nervous system depression, drowsiness, headache, hypotension, mental confusion, nausea, vomiting, rash and nystagmus.

The nurse is caring for a client diagnosed with heart failure who will begin treatment with digoxin. Which therapeutic effect would the nurse expect to find after administering this medication? ADecreased chest pain with decreased blood pressure BIncreased heart rate with increased respirations CImproved respiratory status with increased urinary output DDiaphoresis with decreased urinary output

CImproved respiratory status with increased urinary output Rationale: Digoxin (Lanoxin), a cardiac glycoside, is used in clients with heart failure to slow and strengthen the heartbeat. As cardiac output is improved, renal perfusion is improved and urinary output increases. The other findings are related to adverse, not therapeutic, effects related to digoxin or are not typically seen at all with digoxin.

The nurse is caring for a client prescribed furosemide and digoxin for the treatment of heart failure. The client reports seeing halos and bright lights. Which laboratory result would be anticipated? ALow sodium level BLow digitalis level CLow potassium level DLow serum osmolality

CLow potassium level Rationale: Digitalis toxicity is an accumulation of digitalis (digoxin) in the body that leads to nausea, vomiting, visual disturbances, atrial or ventricular tachydysrhythmias, ventricular fibrillation, sinoatrial block, and atrioventricular block. Clients with heart failure who take digoxin are commonly given diuretics. Hypokalemia can increase the risk of digitalis toxicity. Digitalis toxicity may also develop in the presence of hypomagnesemia. Clients with dig toxicity would have elevated digoxin levels. Sodium would likely be normal. The serum osmolality would likely be normal or high in a client on a diuretic.

The nursing is preparing to administer phenytoin IV push to a client. The client has dextrose 5% in water infusing continuously. Which action is appropriate? APinch the line above the infusion port during the administration BHold the medication and collaborate with the provider prior to administration CStop the infusion and flush the port with normal saline prior to administration DAsk the pharmacy to mix the medication into an IV piggyback (IVPB) infusion

CStop the infusion and flush the port with normal saline prior to administration Rationale: If giving phenytoin as an infusion, it cannot be administered with D5W because it will precipitate. The D5W should be disconnected, the port flushed with normal saline solution (NSS), medication administered, and the port flushed again with NSS before the D5W is reconnected. The provider does not need to be contacted as this is best practice and aligns with hospital protocol. Administering the medication via IVPB does not reduce the risk for

During the admission process, the client reports heavy alcohol use for at least one year. What effect does the nurse anticipate the hospitalized client will experience when alcohol consumption stops? ABradycardia BSomnolence CWithdrawal DTachypnea

CWithdrawal Rationale: The findings of alcohol withdrawal develop within 24 to 48 hours after people either stop or significantly reduce their alcohol consumption. Findings of withdrawal can range from "mild" (shaking or sweating, or perhaps nausea, headache, anxiety, tachycardia or hypertension) to severe (delirium tremens or DTs), which are characterized by rapid heartbeat, fever, hallucinations or seizures.

Which clinical finding indicates that a client taking digoxin may have developed digoxin toxicity? Constipation Decreased urination Cardiac dysrhythmias Metallic taste in the mouth

Cardiac dysrhythmias Rationale The development of cardiac dysrhythmias is often a sign of digoxin toxicity. Constipation is not a sign of toxicity; gastrointestinal signs and symptoms of toxicity include anorexia, nausea, vomiting, and diarrhea. Decreased urination is not a sign of toxicity. Digoxin does not cause a metallic taste in the mouth.

A client receives a cardiac glycoside, a diuretic, an angiotensin-converting enzyme (ACE) inhibitor, and a vasodilator. The client's apical pulse rate is 44 beats/minute. The nurse concludes that the decreased heart rate is caused by which medication? Diuretic Vasodilator ACE inhibitor Cardiac glycoside

Cardiac glycoside Rationale A cardiac glycoside such as digoxin decreases the conduction speed within the myocardium and slows the heart rate. The primary effect of a diuretic is on the kidneys, not the heart; it may reduce the blood pressure, not the heart rate. A vasodilator can cause tachycardia, not bradycardia, which is an adverse effect. ACE inhibitors act on the renin-angiotensin system and are not associated with decreased heart rates.

A nurse is providing education on the use of carbidopa/levodopa to a client with Parkinson's disease. What will the nurse include in the teaching? AThis medication will stop the progression of your condition BNotify your healthcare provider if your urine appears dark CEat plenty of whole-grain foods when taking this medication DAvoid eating meals that are high in protein

DAvoid eating meals that are high in protein Rationale: Carbidopa/levodopa is a combination medication used in the management of Parkinson's disease. Consuming high-protein meals can impair the effects of levodopa. The nurse should instruct the client to eat protein in small portions. Carbidopa/levodopa does not halt the progression of Parkinson's disease. The medication is intended to reduce the symptoms associated with the condition. Darkening of bodily fluids can occur when taking the medication. However, the client should be informed this is not a harmful side effect. Whole grains contain pyridoxine, a vitamin that interferes with the effects of levodopa.

The nurse is caring for a client who has been prescribed atropine preoperatively. The nurse understands the intended purpose for administering this preoperatively is to induce which effect? AReduce heart rate BElevate blood pressure CEnhance sedation DDecrease secretions

DDecrease secretions Rationale: Atropine is a common anesthesia adjunct. It decreases the amount of secretions which, in turn, decreases the risk of aspiration during the operative procedure.

A nurse is preparing to administer prescribed maintenance dose of digoxin to a client who has heart failure. Which action should the nurse to take? AWithhold the medication if the heart rate is above 100/min BInstruct the client to eat foods that are low in potassium CMeasure apical pulse rate for 30 seconds before administration DEvaluate the client for nausea, vomiting, and anorexia

DEvaluate the client for nausea, vomiting, and anorexia Rationale: A client with heart failure who is prescribed digoxin should be assessed for digoxin toxicity. Manifestations of digoxin toxicity include nausea, vomiting, and anorexia. Digoxin is used to decrease heart rate and should be held if the heart rate is less than 60 beats per minute. Digoxin toxicity can occur when the client has low potassium. When administering digoxin, the nurse should measure the client's apical pulse for a full minute.

A child being treated with cardiac medications developed vomiting, bradycardia, anorexia, and dysrhythmias. The nurse understands which medication toxicity is responsible for these symptoms? Digoxin Nesiritide Dobutamine Spironolactone

Digoxin Rationale Digoxin helps improve pumping efficacy of the heart, but an overdose can cause toxicity leading to nausea, vomiting, bradycardia, anorexia, and dysrhythmias. The side effects of nesiritide may include effects such as headache, insomnia, and hypotension. Dobutamine does not cause nausea or vomiting but may cause hypertension and hypotension. Spironolactone may cause edema.

A client who takes multiple medications complains of severe nausea, and the client's heartbeat is irregular and slow. The nurse determines that these signs and symptoms are toxic effects of which medication? Digoxin Captopril Furosemide Morphine sulfate

Digoxin Rationale Signs of digoxin toxicity include cardiac dysrhythmias, anorexia, nausea, vomiting, and visual disturbances. Although nausea and heart block may occur with captopril, these symptoms rarely are seen; drowsiness and central nervous system disturbances are more common. Toxic effects of morphine are slow, deep respirations, stupor, and constricted pupils; nausea is a side effect, not a toxic effect. Toxic effects of furosemide are renal failure, blood dyscrasias, and loss of hearing.

The client with hypokalemia reports nausea, vomiting, and seeing a yellow light around objects. Which of the client's medications is the likely cause of the client's symptoms? Digoxin Furosemide Propranolol Spironolactone

Digoxin Rationale These are signs of digitalis toxicity, which is more likely to occur in the presence of hypokalemia. Although furosemide most likely contributed to the hypokalemia, the client's symptoms are consistent with digitalis toxicity. Although propranolol can cause nausea, vomiting, and blurred vision, the presence of hypokalemia and yellow vision are more suggestive of digitalis toxicity. A side effect of spironolactone is hyperkalemia, not hypokalemia.

One week after being hospitalized for an acute myocardial infarction, a client reports nausea and loss of appetite. Which of the client's prescribed medications would be withheld and the health care provider notified? Digoxin Propranolol Furosemide Spironolactone

Digoxin Rationale Toxic levels of digoxin stimulate the medullary chemoreceptor trigger zone, resulting in anorexia, nausea, and vomiting. Although anorexia, nausea, and vomiting may be side effects of furosemide, propranolol, and spironolactone, they do not indicate toxicity.

The home care nurse is reviewing the medical record of a new client with a history of chronic obstructive pulmonary disease, atrial fibrillation and gout. After reviewing the client's medication list, for which medications should the nurse arrange to monitor blood levels? Select all that apply. Beclomethasone Digoxin Theophylline Allopurinol Montelukast

Digoxin Theophylline Rationale: It is necessary to monitor blood levels for theophylline and digoxin to prevent toxicity. Both of those drugs can accumulate in the blood and reach toxic levels. The other medications are not known to accumulate and cause toxicity if taken as prescribed.

A client has been given a prescription for furosemide 40 mg every day in conjunction with digoxin. Which concern would prompt the nurse to ask the health care provider about potassium supplements? -Digoxin causes significant potassium depletion. -The liver destroys potassium as digoxin is detoxified. -Lasix requires adequate serum potassium to promote diuresis. -Digoxin toxicity occurs rapidly in the presence of hypokalemia.

Digoxin toxicity occurs rapidly in the presence of hypokalemia. Rationale Furosemide promotes potassium excretion, and low potassium (hypokalemia) increases cardiac excitability. Digoxin is more likely to cause dysrhythmias when potassium is low. Digoxin does not affect potassium excretion. Furosemide causes potassium excretion. Potassium is excreted by the kidneys, not destroyed by the liver. Furosemide causes diuresis and consequent potassium loss regardless of the serum potassium level.

The nurse is teaching a client about tricyclic antidepressants. Which potential side effects would the nurse include? Select all that apply. One, some, or all responses may be correct. -Dry mouth -Drowsiness -Constipation -Severe hypertension -Orthostatic hypotension

Dry mouth -Drowsiness -Constipation -Orthostatic hypotension Rationale Dry mouth is a common anticholinergic side effect of tricyclic antidepressants. Drowsiness can be a common side effect but usually decreases with continued treatment. Constipation is a common side effect that usually can be managed with stool softeners and a high-fiber diet. Orthostatic hypotension is a common side effect of tricyclic antidepressants; the client should be instructed to rise slowly from a sitting to a standing position. Hypertension of any type is not a side effect of tricyclic antidepressants.

Which statement accurately describes nortriptyline? Select all that apply. One, some, or all responses may be correct. -Overdosage is often lethal. -Constipation and urinary retention may occur. -It is a selective serotonin reuptake inhibitor (SSRI). -Weight gain is a common side effect. -It increases effectiveness of monoamine oxidase inhibitors (MAOIs).

Overdosage is often lethal. -Constipation and urinary retention may occur. -Weight gain is a common side effect. Rationale: Nortriptyline is notoriously lethal; between 70% and 80% of people who die from overdose do so before reaching the hospital. Nortriptyline can cause constipation and urinary retention because it causes blockage of cholinergic receptors, and it tends to increase appetite and cause weight gain. Nortriptyline is not an SSRI; it is a tricyclic antidepressant and affects the balance of neurotransmitters in the brain. The interaction between nortriptyline and MAOIs is severe and possibly fatal.

A client is prescribed furosemide and digoxin for heart failure. The nurse should monitor the client for which potential adverse drug effect? APulmonary hypertension BAcute arterial occlusion CAcute kidney injury DCardiac dysrhythmias

Question Explanation Rationale: Digoxin is a cardiac glycoside, or positive inotrope that increases myocardial contractility. By increasing contractile force, digoxin can increase cardiac output in clients with heart failure (HF). Furosemide is a potassium-wasting (loop) diuretic, prescribed to prevent fluid overload in clients with HF. Clients who take furosemide are at risk for developing hypokalemia. Potassium ions compete with digoxin and a low potassium level can cause digoxin toxicity, leading to lethal cardiac dysrhythmias. Therefore, it is imperative that potassium levels be kept within normal range (3.5 to 5 mEq/L) while taking digoxin.

A client with heart failure is to receive digoxin. Which therapeutic effect is associated with this medication? -Reduces edema -Increases cardiac conduction -Increases rate of ventricular contractions -Slows and strengthens cardiac contractions

Slows and strengthens cardiac contractions Rationale Digoxin improves cardiac function by increasing the strength of myocardial contractions (positive inotropic effect) and, by altering the electrophysiological properties of the heart, slows the heart rate (negative chronotropic effect). Digoxin increases the strength of the contractions but decreases the heart rate. Although a reduction in edema may result from the increased blood supply to the kidneys, it is not the reason for administering digoxin. Digoxin decreases, not increases, cardiac impulses through the conduction system of the heart.

A health care provider prescribes digoxin for a client. The nurse teaches the client to be alert for which common early indication of acute digoxin toxicity? Vomiting Urticaria Photophobia Respiratory distress

Vomiting Rationale Nausea, vomiting, anorexia, and abdominal pain are early indications of acute toxicity in approximately 50% of clients who take a cardiac glycoside, such as digoxin. Urticaria is a rare, not common, manifestation of digoxin toxicity. Photophobia is a later, not early, manifestation of digoxin toxicity. Respiratory distress is not directly associated with digoxin toxicity.

617) The nurse is monitoring a 6-year-old child for toxicity precipitated by digoxin. Which sign of digoxin toxicity would the nurse monitor for? Oliguria Vomiting Tachypnea Splenomegaly

Vomiting Rationale Vomiting is a sign of digoxin toxicity in children. Oliguria is associated with renal failure, not toxicity. Tachypnea is associated with heart failure, not toxicity. Splenomegaly is associated with heart failure, specifically right ventricular failure.

When teaching a client about digoxin, which symptom will the nurse include as a reason to withhold the digoxin? Fatigue Yellow vision Persistent hiccups Increased urinary output

Yellow vision Rationale Digoxin toxicity is a common and dangerous effect. Visual disturbances, most notably yellow vision, may be evidence of digoxin toxicity. Fatigue is not a toxic effect of digoxin. Persistent hiccups are not related to digoxin toxicity. An increased urinary output is not a sign of digoxin toxicity; it may be a sign of a therapeutic response to the medication and an improved cardiac output.


Kaugnay na mga set ng pag-aaral

Moudule 11~lesson 1 South America🍍🍦📖

View Set

5.7 understand the changes that occur when a solid melts to form a liquid, and when a liquid evaporates or boils to form a gas

View Set

Data Types and the Power of Databases (Practice Test)

View Set